Космос и астрономия


Ответить в тред Ответить в тред

<<
Назад | Вниз | Каталог | Обновить тред | Автообновление
522 47 118

Тред тупых вопросов №122 Михайло Ломоносов эдишон Аноним 05/02/20 Срд 23:36:14 5476341
MikhailLomonoso[...].jpg (2676Кб, 1337x1850)
1337x1850
800px-Lomonosov[...].jpg (195Кб, 800x1123)
800x1123
Herschel-Lomono[...].png (65Кб, 1280x658)
1280x658
Потому что а почему бы и нет?

Тред вопросов о жизни, Вселенной и всём таком.

Спрашиваем то, за что в других местах выдают путёвку в биореактор. Здесь анонимные учёные мирового уровня критически рассмотрят любые гениальные идеи и нарисованные в Paint схемы.

Предыдущий тут: >>543541 (OP)

Q: Можно быстрее?
A: Можно упасть в Пузырь Альбукерке, NASA уже почти надула его.

Q: Я начитался охуительных историй про уфологию, че делать, нам жопа?
A: Да, тебе жопа, можешь сгонять в зогач или куда оттуда пошлют.

Q: Что будет с человеком в вакууме без скафандра / если он упадет на черную дыру / попробует ступить на поверхность газового гиганта/солнца?
A: Он умрёт.

Q: Почему бы не привязать ракету к воздушному шару или стартовать с горы?
A: Космос - это не как высоко, а как быстро, большая часть энергии ракеты уходит на разгон вбок.
Подробнее тут https://what-if.xkcd.com/58/ (английский) https://chtoes.li/orbital-speed/ (перевод)
Аноним 05/02/20 Срд 23:44:49 5476372
почему на Меркурии нету озёр из лавы и атмосферы из горячего газа? Почему он не сплющивается у полюсов, он же нагревается пиздец
Аноним 05/02/20 Срд 23:57:16 5476383
>>547637
Он же железный, что ему сделается-то. И атмосферу всю сдуло давно уже
Аноним 05/02/20 Срд 23:58:34 5476394
>>547637
>озёр из лавы
Там температура максимум 427 градусов по викиданным, маловато для лавы.
>атмосферы из горячего газа
Она вся улетучилась давно. Газ горячий - частицы быстрее, легче улетают. Тяготение маленькое. Еще и солнечный ветер сдувает. Да и само Солнце относительно близко, значит сфера Хилла маленькая, а атмосфера (включая внешние разреженные слои) может быть только внутри нее.
> Почему он не сплющивается у полюсов, он же нагревается пиздец
Как это связано вообще, по-твоему?
05/02/20 Срд 23:59:44 5476415
Опять говноперекат без освящения. Идите нахуй.
Аноним 06/02/20 Чтв 01:05:49 5476456
>>547601 →
Звук это упругие колебания среды, которые распространяются в воздухе, но не распространяются в вакууме. Нет материи — нет и ее колебаний.

Пропадает звук в тот момент, как только ты теряешь контакт со станцией. Прислонившись снаружи шлемом скафандра к стенке, можно будет слышать, что происходит внутри, отодвинешься хотя бы на миллиметр — полная тишина.
Аноним 06/02/20 Чтв 07:35:42 5476557
>>547645
А когда была выдвинута гипотеза и доказано, что звука в космосе нет? Собственно так же, что и воздуха там нет. Это же в какой то момент кто то должен был сначала на теоретическом уровне предположить, а потом и доказали на практике.
Аноним 06/02/20 Чтв 07:54:55 5476568
>>547634 (OP)
Стыдоба сношающаяся!
Во первых абкульерро
Во вторых не надуть а натянуть на корабль, иначе синематографа нет.
В третьих наса контролирует интерференцию сверхточными лазерами, а Луна продолжает отдалятся из-за приливного взаимодействия.
Аноним 06/02/20 Чтв 10:50:52 5476639
>>547634 (OP)
Спасибо, что наконец починили шапку.

Как работает магнетрон? Как микроволновое излучение в целом работает и появляется?
Есть ли звезды у которых основная мощность в микроволновом спектре?
Аноним 06/02/20 Чтв 12:21:24 54766810
>>547663
Есть. Даже мазерные звезды есть
Аноним 06/02/20 Чтв 12:22:51 54766911
>>547639
Могло бы вулканизмом выбрасывать лаву из недр. Странно, что вулканизм слабенький. С такой-то орбитой
Аноним 06/02/20 Чтв 12:37:19 54767212
>>547668
А как микроволны генерятся-то, я это не пойму.
В случае со звездами еще как-то понятно что атомы настолько быстро колбасит что такая частота излучения, а вот как магнетрон работает - вообще не понимаю.
Алсо если у магнетаров не такая природа то тоже буду признателен узнать как на самом деле.
Аноним 06/02/20 Чтв 12:43:27 54767413
1580982206696.jpg (629Кб, 1772x1271)
1772x1271
Какие есть технологии ближайшего будущего (~50 лет), чтобы самолёт мог сам выходить на орбиту?

Может, клиновоздушный двигатель из йоба-сплава?

Просто представляю, как было бы круто, если бы военно-воздушные силы полноправно можно было назвать военно-космическими.

Зарождение славных традиций космофлота!
Аноним 06/02/20 Чтв 12:48:10 54767714
Аноним 06/02/20 Чтв 12:50:11 54767815
>>547674
Осталось придумать задачи для орбитальных самолётов.
Аноним 06/02/20 Чтв 12:51:24 54767916
>>547678
Такие же как и у любых средств вывода на орбиту. Спась Шатал например.
Аноним 06/02/20 Чтв 12:53:34 54768117
>>547678
Попил денег под предлогом защиты от Китая и России
Аноним 06/02/20 Чтв 12:54:12 54768218
>>547681
Поллярда за запуск спасьшатала уже было.
Аноним 06/02/20 Чтв 12:59:24 54768319
>>547678
>задачи для орбитальных самолётов.
Перемещение за полчаса в любую точку планеты.
Неуязвимость для большей части ПВО.
Невидимость для большей части радаров.
Аноним 06/02/20 Чтв 13:02:31 54768420
>>547683
Нет по всем трем пунктам.
Аноним 06/02/20 Чтв 13:04:51 54768521
>>547683
>Перемещение за полчаса в любую точку планеты.
Нахуя?
>Неуязвимость для большей части ПВО.
>Невидимость для большей части радаров.
Сомнительно, но даже если он так далеко что по нему не попадают, что и сам самолет нихуя не сможет сделать, тупо будет на орбите болтаться как говно в ополонке.

Аноним 06/02/20 Чтв 13:10:45 54768722
>>547672
Магнетрон - радиопередатчик, работающий на НУЖНОЙ частоте. Состоит, грубо, из автогенератора на одной вакуумной радиолампе, выход которого практически напрямую подключен к соответствующей частоте направленной антенне, которая в микроволновке находится за "картонкой", на которой, бывает, написано "не протыкать/не рвать".
Аноним 06/02/20 Чтв 13:18:37 54768823
>>547687
Я принцип хочу понять. Я читал статью на википедии, она не помогла мне. Я смотрел видео, не помогло.
Как электрический ток делает ЭМИ микроволновой частоты?
Аноним 06/02/20 Чтв 13:29:48 54769124
>>547688
Так же, как и передатчик в твоём телефоне, роутере, аирподсах, беспроводной колоначке etc. Только на КИЛОВАТТ мощи.
ты дошколёнок?
ты уманетарей совсем?
Аноним 06/02/20 Чтв 13:31:32 54769225
>>547691
Мегагерцы я могу понять, а как гигагерцы получаются, распиши на пальцах.
Да, а тупой уёбок, раз мне вики и ютьюбы не помогли.
Аноним 06/02/20 Чтв 13:41:35 54769526
>>547685
Пикирование в стратосферу, оттуда ракету и обратно в касмас
Аноним 06/02/20 Чтв 13:45:47 54769627
>>547674
прямоточные двигатели до М15
ядерные/термоядерные двигатели
но в любом случае это будет небоевая хуйня с огромными размерами бочек и/или стоимостью реакторов
если и будут военные космолеты, то это будет просто самолет на ракете в стиле dyna-soar из 50х
Аноним 06/02/20 Чтв 13:46:50 54769728
>>547696
Китай не собирается пилить ССТО?
Аноним 06/02/20 Чтв 13:50:31 54769829
>>547697
что-то слышал про ссто с ядерными двигателями, но не ясно есть ли у них хотя бы рендеры этой хуйни. про военное применение не было вроде
Аноним 06/02/20 Чтв 14:22:15 54770030
>>547683
>Перемещение за полчаса в любую точку планеты.

Это так не работает.

>Неуязвимость для большей части ПВО.
>Невидимость для большей части радаров.

Тащемта, на низкой орбите его в оптику видно будет.

>>547695
>Пикирование в стратосферу, оттуда ракету и обратно в касмас

И на сколько таких манёвров ему ресурса хватит? МБР не дешевле будет?
Аноним 06/02/20 Чтв 14:26:50 54770131
>>547678
Кроме ремонта хабблов ничего на ум не приходит.
Аноним 06/02/20 Чтв 14:27:42 54770232
>>547701
Вывод ПН. В автономном режиме в том числе.
Аноним 06/02/20 Чтв 14:50:26 54770533
>>547700
>И на сколько таких манёвров ему ресурса хватит?
На один
>МБР не дешевле будет?
Я вообще хз зачем нужны самолёты-истребители-бомбардировщики в современном мире, когда есть ракеты.
Аноним 06/02/20 Чтв 16:29:01 54771134
>>547634 (OP)
Инструменты для откручивания шурупов и забивания гвоздей в космосе должны быть какими-то особенными, БЕЗЫНЕРЦИОННЫМИ? Что это вообще значит, безынерционный? И какими методами это достигается?
Аноним 06/02/20 Чтв 16:33:53 54771235
>>547711
компенсация сил. например ты включил шуруповерт и он начал вращать тебя вместо шурупа так как невесомость.
Аноним 06/02/20 Чтв 16:43:45 54771336
>>547711
>какими методами это достигается
Вообще, космонавты на станции в эти дни напоминают мешочников. Для работы им нужен инструмент. Ключи, отвертки, молоток, ножницы, набор гаек и болтов. Все это нельзя как в гараже разложить на полу. Разлетится во все стороны и не найдешь потом. Приходится все необходимое таскать с собой. Пристегивать на веревочках капроновых к браслету на руке или к поясу. Мелкие детали обязательно собираются в мешочки, которые тоже надо пристегнуть к себе. По мере работы инструмент отстегивается, мелкие детали выуживаются аккуратно из мешочков. Затем также аккуратно все возвращается на свои места.
Инструмент, которым пользуются космонавты, тоже отличается от земных. И это отличие определяется в первую очередь влиянием невесомости на процесс работы с этим инструментом.
Молоток. Безынерционный. В массив полого молотка вложены металлические шарики. В результате при ударе молоток не отскакивает, а как бы прилипает к поверхности в момент удара. Сила удара сохраняется.
Отвертки анкерные. Все головки болтов, винтов, гаек имеют специальные отверстия. Лезвие отвертки вставляется в прорезь и фиксируется специальным замком. Ручка отвертки имеет специальную скобу. Это позволяет не только работать с гайками и болтами, но и использовать отвертку как надежное фиксирующее устройство. В любом месте космонавт может вставить в крепежное устройство отвертку и зафиксировать положение своего тела для выполнения какой-нибудь работы. Можно использовать анкерные отвертки и для переноски блоков.
Облегчена задача «наживления» винта. На конце резьбовой части имеется цилиндрический хвостовик, диаметр которого равен внутреннему диаметру резьбы.
Сверло. Его диаметр постепенно нарастает от вершины к хвостовику. Это сверл само, без нажима, врезается в материал сверления.
Особой популярностью у космонавтов пользуется безреактивная дрель. Она не передает на руки космонавта реактивные силы и моменты. Кроме того, с помощью сменяемых насадок дрель можно использовать для заворачивания винтов, резки материалов и во многих других случаях.
Аноним 06/02/20 Чтв 16:57:57 54771637
>>547713
>Все это нельзя как в гараже разложить на полу. Разлетится во все стороны и не найдешь потом.
У них же на липучках всё.
Аноним 06/02/20 Чтв 17:02:28 54771738
Аноним 06/02/20 Чтв 19:21:32 54773139
>>547669
Там приливный захват, так что на орбиту плевать
Аноним 06/02/20 Чтв 19:24:06 54773340
15808969945150.jpg (58Кб, 1000x600)
1000x600
>>547634 (OP)
В каких странах диапазон частот этого вашего старлинка уже кем-то застолблён?
Аноним 06/02/20 Чтв 20:10:44 54774141
Сильней ли ёбнет, если две нейтронные звезды столкнутся лоб в лоб, а не крутясь и по касательной, как обычно?
Аноним 06/02/20 Чтв 20:13:43 54774242
>>547741
Да. С гравитационными волнами уносится довольно-таки дохуя энергии.
Аноним 06/02/20 Чтв 20:35:00 54774343
На что интересное можно с биноклем посмотреть из квартиры в Петербурге с окнами только на юг? На Ориона и туманность поглядел, на плеяды и сириуса тоже.
Аноним 06/02/20 Чтв 20:35:28 54774444
>>547743
На соседок напротив.
Аноним 06/02/20 Чтв 21:01:20 54774545
>>547731
Скажи это Энцеладу.
Аноним 06/02/20 Чтв 21:19:38 54774746
>>547731
Из какого года пишешь, из 1970? Приливного захвата там нет, есть резонанс 3:2.
Аноним 06/02/20 Чтв 22:02:16 54775147
>>547747
Бля, вчера же был
Аноним 06/02/20 Чтв 22:56:53 54776348
>>547743
Петербург дохуя засвечен, мало что увидишь.
И окрестности на 200 км вокруг тоже.
Только планетки и созвездия.
Тащемта ничего плохого в созвездиях нет. Учи их, ориентируйся по ним, а как занесет в пердя - уже сможешь глянуть на галактики, например.
Аноним 07/02/20 Птн 01:27:52 54779849
>>547743
>>547744
Тащемта, двачую этого. Всё самое интересное, что ты мог посмотреть, ты уже увидел. С малого надо было начинать. Ну ещё можешь МКС поймать.
Аноним 07/02/20 Птн 02:39:03 54780850
Щас прочитал про экзосферу и экзобазу и у меня появился вопрос: если каким-то образом, создать плотную атмосферу на Марсе или на Луне (я не знаю, обтянув всю планету брезентом и напуская газ под него), то она не улетучится из-за небольшого ускорения свободного падения? Насколько плотной она должна быть например на Марсе, чтобы не улетучиваться?
Аноним 07/02/20 Птн 02:57:39 54781151
>>547674
>Какие есть технологии ближайшего будущего (~50 лет), чтобы самолёт мог сам выходить на орбиту?
Никаких.
Аноним 07/02/20 Птн 03:03:16 54781652
>>547810
За другой интернетовый сленг тоже бугуртишь, маня?
Аноним 07/02/20 Птн 07:10:24 54782953
>>547810
Пидорашка, ты обосралась.
Аноним 07/02/20 Птн 10:28:39 54783954
Будет ли автомат калашникова стрелять на Марсе? на более высокую дистанцию? Там ржавчина хуже чем на Земле работает или на оборот из-за отсутсвия воды и кислорода калаш может 1000 лет в песке пролежать и быть готовым к бою как только ты его поднимешь?
Аноним 07/02/20 Птн 10:33:14 54784155
>>547839
Будет.
На большую дистанцию.
Ржаветь будет сильно дольше, но будет, вода же есть.
Аноним 07/02/20 Птн 10:55:35 54784456
Может ли быть планета из чисто воды, с минимумом камней, или в любом случае железа-силикатов накачает в себя на этапе образования?
Аноним 07/02/20 Птн 11:03:01 54784557
Аноним 07/02/20 Птн 11:08:06 54784658
Красные карлики вообще вещество не разбрасывают, просто догорают и тухнут?
Аноним 07/02/20 Птн 11:24:28 54784859
>>547846
они не догорают, они горят горят горят и может быть в конце жизни горят чуть меньше, но это слишком далеко в будущем
Аноним 07/02/20 Птн 11:26:04 54784960
>>547844
да, накачает, или потом столкнётся с чем-нибудь типа огромного астероида или протопланеты(как Тейя) и оно упадёт в ядро загрязнив сферический шар воды в вакууме
Аноним 07/02/20 Птн 11:27:23 54785061
>>547849
А если гипотетинченски предположить планету из воды. Скажем, какие-то инопланетяне-ебанаты стащили воду с других планет и комет.
Внутри лед будет же, да?
Аноним 07/02/20 Птн 11:30:10 54785162
>>547850
Давление другое будет и там будут экзотические формы льда, да, такие как предполагают внутри Европы, Ганимеда, Каллисто и прочих.
Вода редко бывает чистая, в воде содержатся соли и всякое говно растворённое, даже железо, оно может тоже начнёт медленно оседать в центр?
Аноним 07/02/20 Птн 11:31:28 54785263
>>547851
Интересно.
Спасибо за ответы.
Аноним 07/02/20 Птн 12:24:07 54785864
>>547810
Нюфажек не палится
Аноним 07/02/20 Птн 13:10:36 54786465
>>547808
Она не улетучится из-за брезента
Аноним 07/02/20 Птн 14:39:25 54787166
>>547634 (OP)
Ебанный стыд...
Во-первых, Алькубьерре.
Во-вторых, не упасть, а создавать вокруг корабля изнутри (иначе кина не будет).
В-третьих, НАСА искривляет пространство на десятимиллионную часть, контролируя это сверхточными интерферометрами, до самого варп-привода здесь - как до Антарктиды раком.
Аноним 07/02/20 Птн 15:30:46 54787467
>>547871
>создавать вокруг корабля изнутри

Это невозможно без нарушения принципа причинности.
Если мы хотим, чтобы всё работало, тогда пузырь должен создавать корабль, а не наоборот.
Аноним 07/02/20 Птн 15:39:26 54787568
>>547874
ЯНХНП, объясни подробнее, причем тут причинность?
Аноним 07/02/20 Птн 15:48:18 54787669
>>547875
Не объяснит, он не знает про неё нихуя.
Аноним 07/02/20 Птн 16:28:00 54787770
>>547871
Привет, морская улиточка, хули ты так поздно
Аноним 07/02/20 Птн 21:08:46 54790171
Почему на всякие миссии берут дохуя обученных йоба-астронавтов, делают хуиллиард систем безопасности и прочее? Вот например скорые миссии на Луну. Из 7 миллиардов человек нашлось бы 3-4, которые готовы погибнуть за просто так (например, я). Не получилось? Несите следующих!
Аноним 07/02/20 Птн 21:16:33 54790672
>>547901
Ты сдохнешь и проебешь многомиллиардный аппарат.
Вот поэтому.
Аноним 07/02/20 Птн 23:33:43 54793373
>>547845
Точно так же. Так же и гигагерцевого. Принцип один.
Аноним 08/02/20 Суб 01:14:03 54795674
Выстрелит ли метановое ружье?
Аноним 08/02/20 Суб 02:55:11 54796375
videoplayback.mp4 (243Кб, 320x180, 00:00:07)
320x180
>>547688
>Как электрический ток делает ЭМИ микроволновой частоты?
Аноним 08/02/20 Суб 03:31:45 54796776
>>547743
венеру завтрашним вечером.(ЮЗ)
Аноним 08/02/20 Суб 03:36:15 54796877
>>547967
и альдебаран. близнецы . место где должен был быть млечный путь.

летом еще был сатурн и юпитер чоткие. в aquarius - но щас их уже опустило и развернуло, правда тогда не было ориониуса.
Аноним 08/02/20 Суб 05:54:28 54797878
image.png (383Кб, 689x534)
689x534
>>547634 (OP)
Михаилы, где космоновостей тред?
Аноним 08/02/20 Суб 08:54:47 54798579
14961157759900.png (17Кб, 208x195)
208x195
А разводки для лохов с покупкой участков на Луне, Марсе, с названием разных космических объектов в честь себя любимого, все это еще живо или уже эта сфера наебизнеса отмерла? Помнится даже всякие грамотки и документы выдавали удостоверяющие собственность.
Аноним 08/02/20 Суб 09:31:37 54798780
>>547985
Пиздуй в /biz/, дауна кусок.
Аноним 08/02/20 Суб 09:34:47 54798881
>>547634 (OP)
Наконец-то в шапку альбукерку вернули.
Аноним 08/02/20 Суб 09:35:42 54798982
>>547674
Возвращаемая первая ступень
Аноним 08/02/20 Суб 10:20:35 54799083
>>547963
Ты можешь пояснить или нет?
Аноним 08/02/20 Суб 10:22:25 54799184
>>547933
Я тупой и не понимаю, поясни на пальцах. Тред же для этого.
Аноним 08/02/20 Суб 10:22:50 54799285
>>547988
Ебаный стыд. Во-первых, Алькубьерре.
Аноним 08/02/20 Суб 13:09:11 54803086
Рогозин 19.jpg (112Кб, 722x640)
722x640
>>547713
>Это сверл само, без нажима, врезается в материал сверления.
Аноним 08/02/20 Суб 14:07:25 54804587
Аноним 08/02/20 Суб 17:01:23 54809288
>>547634 (OP)
Какой мощности радиоизлучение прёт от солнца? В сравнении с видимым светом
Аноним 08/02/20 Суб 20:40:03 54812489
>>547991
>поясни на пальцах
А вот хуй тебе! Объясняю на хуях.
Представь, что ты быстро-быстро ебёшь воду. Когда ты суёшь свой хуец в воду, он расталкивает молекулы воды во все стороны и образуется волна. А когда ты накачиваешь электроны в антенну и отсасываешь их обратно, в окружающем поле получается такая же нет волна.
А в магнетроне свободные электроны текут от катода к аноду в магнитном поле, причём могут летать только по пределённой траектории. И при этом каждый электрон гоняет волны. Как будто ты окунул хуй в раковину с водой и водишь кругами. Так.
Аноним 08/02/20 Суб 21:26:33 54813290
blackbodysunems[...].gif (13Кб, 732x428)
732x428
>>548092
В сравнении с видимым светом околонулевое, на 10+ порядков слабее, но в абсолютных числах все равно немалое.
Аноним 08/02/20 Суб 23:05:28 54813991
Взять 2 случая:
1) Ракета взлетает вертикально вверх со 100% тяги двигателя и 100% время работы двигателя;
2) Ракета взлетает вертикально вверх с 75% тяги двигателя и время работы двигателя на 20% больше первого случая.
Оба события происходят в плотных слоях атмосферы. Какая из ракет наберет бОльшую высоту в итоге?
Аноним 08/02/20 Суб 23:23:31 54814692
>>548139
Зависит и от гравитации, и от мощности двигателя, и от плотности атмосферы, но в случае Земли и существующих космических ракет — первая.
Аноним 08/02/20 Суб 23:25:47 54814793
>>548146
>но в случае Земли и существующих космических ракет — первая.
Это работает в любом случае или только до определенного предела сочетания тяги+массы ракеты+коэффициента аэродин.сопротивления?
Аноним 08/02/20 Суб 23:26:59 54814894
>>548147
Перефразирую: чтобы добиться большей высоты полета, нужно в любом случае увеличивать тягу в ущерб времени работы двигателя?
Аноним 08/02/20 Суб 23:29:54 54814995
>>548148
Нет, не в любом. Очень сильно разгоняться в плотных слоях смысла нет, так как аэродинамическое сопротивление растет примерно как квадрат скорости. Для простоты рассуждений сведи к крайнему случаю: гравитация нулевая, на орбиту тебе мешает выйти только атмосфера.
Аноним 08/02/20 Суб 23:37:16 54815096
>>548149
>так как аэродинамическое сопротивление растет примерно как квадрат скорости.
Да, это так. Но так как время работы двигателя меньше, то ракета и не успеет разогнаться до запредельных скоростей.
Скажем, если поставить условие, что ракета не разовьет более 1000 км/ч.
Аноним 09/02/20 Вск 00:51:15 54815497
800px-Fountains[...].jpg (70Кб, 800x678)
800x678
Если Энцелад постоянно выбрасывает в космос столько воды, она же должна кончится в конце концов?
Аноним 09/02/20 Вск 01:24:09 54816198
>>548154
>Cryovolcanoes near the south pole shoot geyser-like jets of water vapor, molecular hydrogen, other volatiles, and solid material, including sodium chloride crystals and ice particles, into space, totaling about 200 kg per second.[14][18][20]
Хуйня, тащемта, порядка 6000 тонн в год. Даже если океан весит 0,001% от всего Энцелада, то это все равно около 10^12 тонн. Поступление от падающих комет тоже ненулевое наверняка (редко, но помногу сразу). Даже если он сильно "исчерпается", гейзеры иссякнут задолго до того, как океан совсем все.
09/02/20 Вск 01:53:38 54816499
>>548161
Там он ещё выбрасывает материал, а он обратно возвращается ещё, лол, тусуясь в кольце и потом часто выпадая на поверхность.
Аноним 09/02/20 Вск 03:22:13 548167100
>>548124
Но ведь частота тога не гигарецовая, почему частота микровол ниггагерцовая?
Аноним 09/02/20 Вск 03:23:47 548168101
>>548139
Зависит от ускорения, а не процентов двигателя.
Если дохуя быстро ускоряется, то будет неэффективно из-за торможения об атмосферу.
Если бы атмосферы не було, то наоборот, та что шустрее меньше на гравитацию потеряет.
Самое оптимальное ускорение - моментальное будет.
Аноним 09/02/20 Вск 03:24:49 548169102
image.png (4421Кб, 1392x2073)
1392x2073
>>548154
Связан ли этот спутник с одноименным блюдом?
Аноним 09/02/20 Вск 12:57:13 548190103
>>548169
Нет, ни со спутником, ни с греческой мифологией блюдо не связано.

> Enchilada is the past participle of Spanish enchilar, "to add chili pepper to"; literally, "to season (or decorate) with chili".

Как видишь, этимология совершенно другая.
Аноним 09/02/20 Вск 13:01:34 548191104
>>548190
Космач познавательный.
Аноним 09/02/20 Вск 13:07:21 548192105
>>548191
А то! И швецы, и жнецы, и на анусных ставках игрецы!
Аноним 09/02/20 Вск 13:35:15 548194106
>>548192
Хоть один анус-то выиграли?
Аноним 09/02/20 Вск 13:36:23 548195107
>>548194
Я постоянно выигрывал, это не сложно, если тенденции видишь.
Аноним 09/02/20 Вск 15:06:48 548201108
>>548154
Возможно ли этот спутник лучший вариант для колонизации?
Аноним 09/02/20 Вск 15:21:53 548204109
>>548201
Что ты там колонизировать собрался? Человеки не живут в подземных океанах, им обитаемую поверхность подавай. Для поиска внеземной жизни - возможно. Для колонизации - однозначно нет.
Аноним 09/02/20 Вск 15:22:49 548205110
>>548204
>Человеки не живут в подземных океанах
Опа... Так что, все те мужики, которых мы с пацанами выбрасывали в мешках в Гудзон... О, нет...
Аноним 09/02/20 Вск 15:27:15 548206111
>>548161
Нолики потерял. 6 миллионов тонн в год.
Аноним 09/02/20 Вск 16:30:06 548212112
Поясните за сверхмассивные ЧД. Они весят в миллиарды раз больше солнца. Но так как ЧД появилась из взорвавшейся звезды, то сколько же эта звезда весила? Как триллион солнц?
Аноним 09/02/20 Вск 16:32:34 548215113
>>548212
Не, это куча материи много много лет падала в какую-нибудь первичную ЧД, либо быстро за миллион лет сгоревшую массивную (в 150 масс солнца, больше не бывает), либо первичную промежуточного калибра в тысячи масс, либо что-то посередке в виде кучи дыр народившихся поначалу склеившихся друг с другом. Должны быть охуительные отголоски такого поглощения, по идее.
А потом миллионы и миллиарды лет падающего говна, распидорашиваемых звезд и мимокрокодивших чд тормозивших о диск.
Аноним 09/02/20 Вск 16:35:01 548218114
>>548215
>150 масс солнца
Это просто пылинка, даже не пылинка, а одна молекула по сравнению с миллиардом масс солнца.
Аноним 09/02/20 Вск 16:36:08 548220115
>>548218
Хорошо, что у тебя целая галактика этого говна вокруг и СМЧД не образуется за несколько лет, верно?
Аноним 09/02/20 Вск 22:57:57 548277116
>>548220
И чо, вся материя в пределах галактики в итоге схлопнется в одну чд?
Аноним 09/02/20 Вск 22:59:58 548279117
>>548277
Нет, окраины могут остаться черными карликами и черными дырами звездных масс которые излучатся хокингом.
А вот центр за миллиарды и триллионы лет попадает. Зависит от галактики, конечно.
Аноним 10/02/20 Пнд 12:58:17 548332118
>>547634 (OP)
Чем отличается тундра от молнии?
Аноним 10/02/20 Пнд 13:01:32 548333119
>>548332
Тундра - это климатическая зона, а молния - природное явление.
Тундра геосинхронная и висит над одной точкой большую часть дня. Молния - обычно по полдня и предназначена работать над полушарием, на высоких наклонениях. Обе штуки по одиночке для коммуникаций не заюзать, надо группировку на тех же орбитах.
Аноним 10/02/20 Пнд 13:26:27 548337120
>>548332
Примерно тем же, чем крокодил от зонтика.
Аноним 10/02/20 Пнд 15:45:08 548350121
mislivirazheniy[...].jpg (42Кб, 594x358)
594x358
На расстоянии скольких километров будет находиться край эргосферы от стандартной, 10 километровой черной дыры?
Аноним 10/02/20 Пнд 18:51:53 548368122
>>548350
>стандартной
По какому госту?
Аноним 10/02/20 Пнд 19:00:21 548371123
>>548368
ГОСТ Р-Ч3-2.2093
Вращающаяся незаряженная черная дыра звездной массы.
Аноним 10/02/20 Пнд 19:05:04 548372124
>>548371
Вращаться можно с разной скоростью. Можно как карусель, а можно как вал фрезерного станка.
Аноним 10/02/20 Пнд 20:21:46 548388125
>>548372
А можно намотаться на чёрную дыру?
Аноним 10/02/20 Пнд 21:53:35 548407126
>>548388
Только если Хокинг уйдёт с площадки
Аноним 10/02/20 Пнд 21:59:18 548410127
>>548388
Ну ты спагеттифицируешься, а спагетти наматывать вполне реально, так что при некоторых условиях - можно, разрешаю.
Доложите об исполнении.
Аноним 11/02/20 Втр 02:34:51 548434128
>>547864
А если мгновенно убрать брезент?
Аноним 11/02/20 Втр 02:42:54 548435129
Во что превратится человек упавший на поверхность Венеры?
Аноним 11/02/20 Втр 02:44:01 548436130
>>548435
И да, если случайно столкнуть кого-нибудь на Венеру, все будут считать это несчастным случаем, потому что доказательств нет?
Аноним 11/02/20 Втр 02:59:26 548438131
>>548435
>>548436
Смотря откуда и как.
Если сойдет с орбиты - то сгорит нахуй. То же, что и у нас, тащемта.
Аноним 11/02/20 Втр 03:12:39 548439132
>>548434
Атмосфера начнет растягиваться в высоту, пока график ее плотности не примет форму естественной экспоненты с соответствующей scale height (не знаю, как по-русски называется этот термин).

Верхние слои, разумеется, тут же понемногу начнут улетучиваться, но это не быстрый процесс, даже ополовинивание атмосферы займет десятки миллионов лет.
Аноним 11/02/20 Втр 04:31:58 548440133
Аноны, минут 25 назад в Омске, в направлении северо-востока, видел как что-то развалилось в атмосфере и сгорело. Что это было? Чей-то спутник упал?
Аноним 11/02/20 Втр 04:39:44 548441134
>>548167
Микроволны - результат собственных колебаний в электронном облаке, которое заперто в резонаторной камере. У тебя есть ведущая НЧ электромагнитная волна, есть камера с коэффициентом резонации. Ведущая волна отклоняет электронное облако, не дает ему передать энергию аноду, из-за чего это запертое облако начинает генерировать на высокой частоте - дробовой шум. Этот шум и есть гигагерцы. Регулированием частоты ведущей волны и конфигурацией резонаторной камеры меняют частоту магнетрона. Сами электроны в камеру поступают от постоянного тока, кстати.
Аноним 11/02/20 Втр 04:42:58 548442135
Аноним 11/02/20 Втр 04:48:33 548443136
image.png (248Кб, 400x311)
400x311
>>548441
Заебись, спасибо!
Аноним 11/02/20 Втр 05:21:06 548446137
Без имени.png (360Кб, 925x641)
925x641
>>548440
Немного более понятная схема. Чёрным — направление моего взгляда, красным — примерное направление движения объекта
Аноним 11/02/20 Втр 05:32:54 548448138
>>548440
Если тебе не показалось и это правда был какой-то космический объект, то это скорее всего был обычный метеор.

Спутники входят в атмосферу менее чем раз в сутки (и то обычно над океанами), а естественные камешки, достаточно крупные, чтобы их было видно глазом — тысячами.
Аноним 11/02/20 Втр 05:35:20 548449139
image.png (181Кб, 300x300)
300x300
>>548448
Я бы еще не стал исключать из внимания тот факт, что он из Омска, но да, метеор - самое вероятное. Они как-то ради МКС не должны отслеживаться, часом?
Аноним 11/02/20 Втр 05:38:31 548450140
>>548448
>Если тебе не показалось
Я отчётливо видел яркий светящийся объект, потом от него отделилось несколько объектов меньше и они все стухли через секунду.
Аноним 11/02/20 Втр 05:47:33 548451141
>>548449
А как их отследить? Большинство видимых метеоритов это камешки массой всего в несколько грамм, движущиеся на 30+ км/с, их совершенно невозможно заметить с поверхности. Да даже хуйню весом в тонны надежно засечь хотя бы за минуты до входа в атмосферу не представляется возможным — слишком уж маленькая и слишком быстро движется.

Реально отслеживаются только околоземные астероиды от сотни метров в диаметре.
Аноним 11/02/20 Втр 06:12:47 548453142
>>548449
>>548448
Страшный пиздец на самом деле. Пожалуй, стоит добавить в "космические страхи". Сычуешь ты такой на МКС, и тут в тебя прилетает камушек. Он вроде и небольшой, 5-10кг всего, но такого камушка будет достаточно, чтобы необратимо распидорасить целый модуль, а может и порвать МКС на две половинки. И кто знает, сколько таких камушков тусуется на опасных орбитах пересечения с Землёй.
Аноним 11/02/20 Втр 15:00:40 548482143
>>548453
Вероятность столкновения с другим спутником или просто мусором (обломки, типа), скорее всего, выше. В конце концов, спутники уже сталкивались, а вот метеоритом вроде как ни один не прибило.

https://ru.wikipedia.org/wiki/%D0%A1%D1%82%D0%BE%D0%BB%D0%BA%D0%BD%D0%BE%D0%B2%D0%B5%D0%BD%D0%B8%D0%B5_%D1%81%D0%BF%D1%83%D1%82%D0%BD%D0%B8%D0%BA%D0%BE%D0%B2_%D0%9A%D0%BE%D1%81%D0%BC%D0%BE%D1%81-2251_%D0%B8_Iridium_33
Аноним 11/02/20 Втр 19:40:00 548502144
Мы сейчас про метеоры говорили. А они же сгорают не с орбиты, а набижав где-то под большим углом, вне земной орбиты.
Мелочь отследить там с земли и нереально по идее.
А какого размера они обычно, и какой ущерб могут нанести при попадании в МКС, их же не предупредить
Аноним 11/02/20 Втр 21:12:54 548534145
Известно, что от гаммы защищают электроны, чем больше их, тем меньше гаммы пройдет, именно поэтому на единицу объема лучше всего подходят атомы дальше в таблице.
А что будет если ты поставишь между источником гаммы (в вакууме) и детектором (в вакууме) метровую стену состояющую из максимально положительно заряженного свинца (в вакууме)?
Что если наоборот?

Или изначальная предопсылка неверна?
Аноним 12/02/20 Срд 02:03:50 548558146
>>548534
Вангну, что разница будет минимальная. Вряд ли там существенно отличное количество электронов по сравнению с нейтральным состоянием.
Аноним 12/02/20 Срд 06:18:05 548564147
По самым оптимистичным прогнозам - ядерные реакторы в космосе когда?
Аноним 12/02/20 Срд 07:38:37 548565148
Аноним 12/02/20 Срд 07:39:49 548566149
>>548565
Ха-ха, но нет, я имею в виду реально когда сможет полететь, в прошлом уже не сможет.
Аноним 12/02/20 Срд 08:53:20 548568150
>>548564
> По самым оптимистичным прогнозам - ядерные реакторы в космосе когда?
Когда VASIMR допилят.

А вообще конкретизируй, что ты именно подразумеваешь под реактором. РИТЭГи и так летают. Мегаваттники пока возить не на чем и незачем - для начала надо хотя бы ныне разрабатываемые сверхтяжи до ума довести и начать хотя бы что-то лунное. ЯРД/ГФЯРД вообще непонятно, куда и на какие шиши засовывать.

Все эти мега ёбы нужны для космической экспансии, а она упирается в совсем другие вещи. Пока ещё не ясно, до каких цифр можно опустить стоимость килограмма на НОО, насколько оправдана крупномасштабная сборка на орбите, с каких пропорций кориолисовы силы перестанут ебать искуственную гравитацию, насколько страшна проблема радиации и что делать, если в таком полёте что-то пойдёт не так. Это первичные вопросы, их будут отрабатывать на LOP-G, на бигелоу модулях всяких, пока общая концепция дальнего межпланетного огурцевоза не нарисуется.
Аноним 12/02/20 Срд 08:53:21 548569151
>>548566
Может в любой момент (ну плюс год на постройку), просто нет запроса. С улучшением характеристик аккумуляторов и солнечных панелей реакторы стали тупо не нужны, вплоть до орбиты Юпитера можно работать на солнечной энергии, а дальше все равно ничего и не летает. Марсоходам и луноходам хватает и РИТЭГов.

Реально реактор снова понадобится тогда, когда будут люди на поверхности Марса или Луны, а когда они там будут, никто тебе точно не скажет. Наверное, где-то около 2030 года, не раньше.
Аноним 12/02/20 Срд 10:28:53 548570152
>>548569
>Марса
>около 2030 года
Я бы этим смертникам и еды не давал, тем более реакторы. Прилетели, высадились для галочки и сдохли через неделю.

А реакторы уже с роботами муравьями запускал.
Аноним 12/02/20 Срд 12:03:15 548577153
140781770730072[...].jpg (285Кб, 1600x1130)
1600x1130
>>548436
Конечно
>>548534
>Известно
Кому, блядь, изветсно? тебе? Электроны ему защищают. Ты вообще в курсе, что максимум можешь увеличить концентрацию электронов процентов на 10-18, а потом (а может и раньше) твою стену распидарасит. И да, электроны отрицательно заряжены.
Аноним 12/02/20 Срд 12:36:46 548581154
>>548139
Процент тяги не имеет значения. Просто есть определенный расчет: Выше такой-то скорости при данной плотности атмосферы нет смысла разгоняться, т.к. сопротивление воздуха пропорционально квадрату скорости. Поэтому берут самый легкий двигатель, который может достичь желаемой скорости... Естественно есть еще куча факторов: максимальное ускорение, топливоэффективность на разных высотах и в вакууме.

В общем, тяга на 100% для того, что если тяга не на 100%, то нахуя тогда такой тяжелый двигатель? Давайте сделаем полегче, и тягу на 100%
Аноним 12/02/20 Срд 12:53:18 548584155
>>548534
>>548577
Я в своё время более прогрессивную штуку придумал.

Суть: конденсатор очень малой ёмкости (пикофарады) и очень высокого напряжения (мегавольты). Поскольку E=CU^2/2, то даже изменение напряжения на долю вольта потребует очень солидного вливания энергии, и эта энергия может превысить энергию гамма-кванта. Проще говоря, ионизировать такой конденсатор, пробив его вдоль, мешает злое электростатическое поле.
Далее, борзый гамма-квант сначала попадает в стенку, вызывая ливень вторичных частиц, они уже простреливают конденсатор насквозь. Но поскольку конденсатор охуенно заряжен - вторичные частицы убивают об него всю свою энергию, и дальше уже не летят. А чтобы конденсатор не зарядился до пробоя, мы понемногу сливаем с него энергию через каскад трансформаторов, получая в довесок хоть слабенький, но источник энергии.
Аноним 12/02/20 Срд 15:36:34 548603156
>>548577
Thunderfoot сказал здесь, он физик и шарит. https://youtu.be/568iDYn8pjc?t=339 Диспруфай его или иди на хуй.
>И да, электроны отрицательно заряжены.
И к чему ты пизданул, умник?
Нихуя не шаришь, лишь бы доебаться, свободен.

Аноним 12/02/20 Срд 16:18:28 548612157
>>548603
>нит ита ни абасрался а ты!!!!
Аноним 12/02/20 Срд 16:20:18 548613158
>>548612
>зеленый пук вместо конструктива
Я же сказал, что нихуя не шаришь, чмоха. Съеби с доски, такие тупые ебланы здесь не должны быть.
Аноним 12/02/20 Срд 16:57:21 548618159
А при ядерном взрыве воздух озонируется?
Если пережить изначальную вспышку и волну и сразу пойти погулять, будет пахнуть озоном? Какие подводные в пешей пробежке куда подальше сразу после удара?

Ну и сразу астрономический вдогонку - метеориты по мощности сопоставимы, а то и превосходят ядерные взрывы. А эти взрывы радиоактивны, или абсолютно безвредны (кроме взрывной волны убивающей)?
Аноним 12/02/20 Срд 17:29:48 548620160
>>548618
Наоборот, от нагрева воздуха высираются всякие оксиды азота, которые потом подымаются в верхние слои и реагируют с атмосферным озоном.

https://doi.org/10.1029/RG013i004p00451
Аноним 12/02/20 Срд 17:32:19 548621161
>>548620
Представил себе дождь из тетраоксида азота падающий на гептиловые поля на каком-нибудь полигоне в казахстане и умилился.
Аноним 12/02/20 Срд 17:36:53 548622162
>>548618
>Какие подводные в пешей пробежке куда подальше сразу после удара?
Наглотаешься вместе с пылью не осевших радиоактивных частиц и умрёшь от рака лёгких/желудка.
Аноним 12/02/20 Срд 17:39:22 548623163
>>548622
Так они еще в облаке же, нет? Я имел в виду сразу после прошествия взрывной волны бечь куда подальше.
Алсо, если наглотаться/надышаться радиоактивных частиц, до до рака не доживешь.
Аноним 12/02/20 Срд 18:27:16 548626164
image.png (1976Кб, 1920x1080)
1920x1080
image.png (1392Кб, 820x1300)
820x1300
Почему на скафандрах эти элементы - ремешочки и прочее - синего цвета? Не был бы оранжевый более заметным?
Есть ли причина для выбора этого цвета?
Аноним 12/02/20 Срд 19:10:57 548629165
12/02/20 Срд 19:19:37 548630166
image.png (1669Кб, 1000x704)
1000x704
image.png (1588Кб, 1020x655)
1020x655
>>548626
Они синие и красные чтобы на снимках легко опознавать члена пары было. Соколы синие наверно потому что ВВС(?)
Аноним 12/02/20 Срд 19:39:55 548632167
>>548630
>Они синие и красные чтобы на снимках легко опознавать члена пары было.
Бля, охуенчик. А если тройной выход, тогда как? Зеленый бы добавили?
Аноним 12/02/20 Срд 20:14:08 548635168
a-space-odyssey[...].jpg (844Кб, 1936x2592)
1936x2592
>>548632
Ну не нравится зелёный - заебошьте жёлтый.
Аноним 12/02/20 Срд 20:16:30 548636169
image.png (90Кб, 281x180)
281x180
>>548635
Я за Ордосов хочу.
Аноним 12/02/20 Срд 20:16:57 548637170
>>548632
Космонавты ни разу втроём вкосмос не выходили, только астронавты.
Аноним 12/02/20 Срд 20:17:51 548638171
>>548637
Я и не говорил, что выходили, я спрашивал, как было бы.
А у астронавтов как?
Аноним 12/02/20 Срд 20:25:27 548641172
>>548638
>А у астронавтов как?
Пару раз всемером вышли.
Аноним 12/02/20 Срд 20:26:09 548642173
>>548641
Цвета, я про цвета хотел узнать.
Аноним 12/02/20 Срд 20:56:43 548649174
В чем проблема голой сингулярности? Приводит к парадоксам?
Аноним 12/02/20 Срд 22:38:32 548662175
>>548641
Это когда такое было? Если это шутка про катастрофы Шаттлов, то ты пидор и там в обоих случаях был не космос.
Аноним 12/02/20 Срд 22:46:20 548664176
laugh cry krist[...].gif (497Кб, 220x220)
220x220
>>548662
Пиздос, до меня и не дошло, про 7 астронавтов.
Аноним 12/02/20 Срд 22:51:14 548665177
>>548662
Ну да, возможно, шутка аморальная и неудачная. Но чё сразу пидор-то?

>>548664
Ебать ты слоу.
Аноним 12/02/20 Срд 22:51:36 548666178
>>548641
Хорош! Я тоже слоу.
Аноним 12/02/20 Срд 22:53:01 548667179
mike really sad.jpg (35Кб, 663x579)
663x579
>>548665
Охуенная шутка, но все равно грустно о таком вспоминать. Космические фраги это болезненно.
Аноним 12/02/20 Срд 23:00:29 548668180
>>548667
>Those who made the ultimate sacrifice so others could reach for the stars
>Ad astra per aspera

F
Аноним 12/02/20 Срд 23:03:18 548669181
Как думаете, индусы, китайцы или иранцы первыми угробят астронавтов следующими? Или же мы реально проживём оставшийся 21 век без единого потерянного астронавта? Было бы круто, но кажется шансы этого слишком низки.
Аноним 12/02/20 Срд 23:07:15 548670182
>>548669
>иранцы
Братан, это случайно не те, что недавно свой первый скафандр показали, а он оказался детским костюмом для хэллоуина с алиэкспресса?
Аноним 12/02/20 Срд 23:08:10 548672183
>>548669
Ну, если космонавтика будет развиваться то к концу века мы точно получим космические аналоги 737 макс инб старлайнер
Аноним 12/02/20 Срд 23:25:06 548676184
>>548668
Знаешь, что особенно смешно? Как американцы умудряются поэтической чушью сменить тему со смерти астронавтов на единение и мечты.
Если в СССР умирает космонавт - "виновата система, просрали, расстрелять, Юра мы всё просрали, совок убивает лучшких, сместить конструктора, кто лично виноват, как его имя???

Если в США умирает астронавт - он был героем, давайте минутку молчания, мы полетим к звёздам, его жертва будет не напрасной, назовём его именем корабль, летс целебрейт хис ачивментс, не забудем. (никто не наказан, никто не обвиняет президента или главу НАСА или конструктора в смерти, все заняты патетическими поэтическими высерками и фантазиями)
Аноним 12/02/20 Срд 23:30:29 548677185
Аноним 12/02/20 Срд 23:37:39 548681186
>>548676
Чет зря ты порвался, по-моему.
Везде траур и везде разбирательства и попытки сделать чтобы подобное не повторялось никогда.
Аноним 13/02/20 Чтв 00:03:08 548689187
>>547634 (OP)
Выпали в ютубе рекомендуемые видосы лекции В Сурдина.
Охуенно рассказывает. Пересмотрел всё с ним. Есть ещё аналоги?
Аноним 13/02/20 Чтв 00:04:47 548691188
Аноним 13/02/20 Чтв 00:32:49 548696189
А я правильно понимаю, что все американские космические корабли запускаемые верхом на ракете были без кожуха? Я про проект ртуть, близнецы, аполлон и грядущие.
Аноним 13/02/20 Чтв 00:48:22 548699190
>>548696
У Меркьюри, Джемини и Аболона достаточно обтекаемая форма и жаропрочное покрытие, поэтому кожух им не нужен.

У Аболонов был кожух на командном модуле, чтобы система спасения реактивными струями не упиздякала иллюминаторы.
Аноним 13/02/20 Чтв 00:51:27 548701191
BlueprintExplod[...].jpg (50Кб, 800x495)
800x495
Orionspacecraft[...].png (492Кб, 1920x1080)
1920x1080
>>548696
Капсула Аполлона была закрыта обтекателем системы САС. У Ориона то же самое, плюс аэродинамические панели на служебном отсеке.
Аноним 13/02/20 Чтв 01:08:09 548703192
image.png (906Кб, 1200x841)
1200x841
>>548699
Так у ртути и близнецов тоже иллюминаторы наружу, у близнецов аж с такой формой, что пламя прям в них и останется, почиму для них не надо было?
САС меньше и не так сильно жарить будет?
Аноним 13/02/20 Чтв 01:20:19 548704193
>>548703
У Джемини нет системы спасения капсулы — там катапультные кресла у астронавтов.
Аноним 13/02/20 Чтв 01:23:50 548705194
>>548704
Тьфу, епт, запамятовал.
Аноним 13/02/20 Чтв 01:24:26 548706195
>>548704
А у Ртути-то что? Окна ж прямо на виду. Или они только потом задумались, что в форточку надуть газиками САС надуть может?
Аноним 13/02/20 Чтв 01:25:37 548707196
g043b.gif (27Кб, 480x417)
480x417
EscaperocketofM[...].jpg (91Кб, 640x823)
640x823
>>548703
У Джемини САС представляла собой катапультируемые кресла, которые вылетали под дружный хохот весельчаков-астронавтов через открывающиеся люки. Вроде кстати кто-то рукой уебался о люк на испытаниях этой системы и был не очень рад полученным ощущениям, но серьёзных травм не получил, но сейчас нагуглить эту кулстори не могу.
Меркурий же оче маленький, башня относительно него довольно высокая, заряды не такие мощные. Да и окно там одно и совсем махонькое.
Аноним 13/02/20 Чтв 01:26:50 548708197
>>548707
Это всё, что я хотел услышать и даже больше.
Спасибо.
Аноним 13/02/20 Чтв 01:29:36 548709198
>>548708
Я в догонку ещё добавлю что поскольку в Меркурии было одно окно, сопла САС можно было разместить так, чтобы реактивные струи в него не попали.
Аноним 13/02/20 Чтв 05:55:25 548716199
>>548703
>>548707
По САС Джемини добавлю, что уже после программы поняли, что она получилась самоубийственной - корабли стартовали с атмосферой их чистого кислорода с давлением чуть ли не выше атмосферного, в котором горит вообще все, экипаж Аполлона-1 не даст соврать. Для пожара достаточно малейшей искры. Срабтывание реактивных поджопных двигателей практически навернякка заставило астронавтов полыхнуть.
Аноним 13/02/20 Чтв 06:08:32 548718200
>>548716
Это же шин.
Одеваешь космонавтов в асбестовые скафандоры, а в катапульту заливаешь гиперголик который от контакта с кислородом срабатывает. Экономия!
Аноним 13/02/20 Чтв 06:08:53 548719201
001.jpg (66Кб, 786x442)
786x442
>>548635
А розовый скафандр для тяночек сделают?
Аноним 13/02/20 Чтв 09:35:02 548722202
>>548701
Это чтобы САС не зажарила капсулу
>>548696
Это скорее советские капсулы исключение из-за ограничений по диаметру и необходимости приделывать бытовой отсек
Аноним 13/02/20 Чтв 10:24:52 548727203
Не возьмет ли меня за жопу товарищ майор, если я выложу на своей страничке ВК видео с запуском моей говноракеты?
Или лучше не рисковать?
Аноним 13/02/20 Чтв 11:32:54 548729204
image.png (2224Кб, 1788x900)
1788x900
>>548630
довен это просто скины разные
Аноним 13/02/20 Чтв 11:35:16 548730205
>>548727
Я не разбираюсь в законах но я бы не рисковал. Потому что сегодня ты выкладываешь и всем ок, а через 5 лет примут закон что запуск любой хуиты выше 2 метров считается изменой родины и тебя нежно берет за яйтса участковый, мечтающий о премии.
Аноним 13/02/20 Чтв 14:41:59 548741206
>>548730
>считается изменой родины
А как это меняет родину?
Аноним 13/02/20 Чтв 14:43:02 548742207
>>548741
В выпадающем списке родин выбирается другая.
Аноним 13/02/20 Чтв 15:03:05 548743208
Аноним 13/02/20 Чтв 15:31:20 548748209
>>548730
А как же "закон обратной силы не имеет"?
Аноним 13/02/20 Чтв 15:36:42 548750210
Стикер (63Кб, 250x250)
250x250
>>548748
Ты всерьёз сейчас этот вопрос задаёшь?
Аноним 13/02/20 Чтв 15:39:35 548751211
изображение.png (146Кб, 512x337)
512x337
Аноним 13/02/20 Чтв 15:41:13 548752212
>>548442
The New Order: Last Days of Europe
Аноним 13/02/20 Чтв 15:51:01 548754213
Говорят для защиты от космической радиации достаточно обвесить внутреннюю поверхность корпуса корабля пакетами с водой и другими припасами. А во время солнечных бурь можно прятаться в кабинки с усиленной защитой.
Это правда? И даже свинец не нужен? И никто не подохнет от рака?
Аноним 13/02/20 Чтв 15:51:56 548755214
>>548754
Свинец не нужен.
Это пустой бесполезный вес. Задействуется полезная нагрузка.
Аноним 13/02/20 Чтв 15:53:07 548756215
>>548755
>Задействуется полезная нагрузка.
Она поглощает радиацию эффективнее свинца? Сомневаюсь. Да и как потом жрать фонящую жратву?
Аноним 13/02/20 Чтв 15:54:33 548757216
>>548756
Еще раз - свинец это пустой бесполезный вес.
И жратва не станет фонить.
Аноним 13/02/20 Чтв 15:55:24 548758217
>>548756
даун блядь радиацию поглощает масса
свинец просто наиболее удобное сочетание плотности и цены
Аноним 13/02/20 Чтв 15:57:11 548759218
>>548758
>
>даун блядь радиацию поглощает масса
проиграл с этого мамкиного физика
Аноним 13/02/20 Чтв 16:02:10 548761219
>>548759
Ты весь тред гринтекстом пукаешь и проигрываешь, а нормальной контраргументации не принес.
Да и как ты принесешь, лол, это объективный факт.
Аноним 13/02/20 Чтв 16:28:29 548767220
>>548761
хули толку от твоей массы, если плотность никакая и ядра атомов маленькие?
лень метать бисер и рассказывать про сечение взаимодействия и другие понятия
Аноним 13/02/20 Чтв 16:34:57 548770221
>>548757
Да только свинца надо метровый слой чтобы защитить от космической радиации. Жратвы там сколько будет? Километровый слой что ли?
Аноним 13/02/20 Чтв 16:44:11 548776222
>>548767
То есть водород у тебя не останавливает радиацию?
Еблан, ну что за пиздец.

>>548770
Что значит "защитить"? Надо снизить уровень до безопасных значений.
Если тебе нужен прям метровый слой свинца (не нужен), то жратвы/воды будет достаточно десятиметрового.
А вот если захочешь жидким водородом укрываться, то его понадобится 150 метров.
Аноним 13/02/20 Чтв 17:42:40 548794223
>>548776
> не нужен
На себе испробуешь? А мы посмотрим, что от тебя останется в конце полета.
Аноним 13/02/20 Чтв 18:03:16 548798224
При помощи каких средств человек, будучи в ракете/шаттле/пепелаце, может без особого вреда пересечь пояс радиации? Насколько мне известно, солнечная радиация - это пиздец какая уберхуйня. И каким образом, в таком случае, можно осуществить полёты на Марс?
Аноним 13/02/20 Чтв 18:13:05 548800225
>>548794
Без проблем.
По примерным прикидкам достаточно метрового слоя воды и поворачивать корабль баками к солнцу во время вспышек. За полгода полета даже в случае одного выброса ничего страшного не произойдет.

Попробуй сам посчитать, епт.

Или ты вообще не разбираешься, и считаешь, что только свинец от радиации "защищает", а остальное нет, и "радиация" это одна та неведомая хуйня от которой только свинец, и больше никак.
Аноним 13/02/20 Чтв 18:16:01 548801226
image.png (36Кб, 1534x652)
1534x652
>>548798
>При помощи каких средств человек, будучи в ракете/шаттле/пепелаце, может без особого вреда пересечь пояс радиации?
При помощи полярной траектории, очевидно. Не лететь по экваториальной и нахождение в потоке захваченных заряженных частиц будет минимально.
>Насколько мне известно, солнечная радиация - это пиздец какая уберхуйня.
Да. И благодаря ей существует жизнь.
Заряженные частицы захватываются магнитным полем, а ионизирующее излучение останавливается атмосферой.
>И каким образом, в таком случае, можно осуществить полёты на Марс?
Собрать ракету побольше, хавки на два года, и полететь по гомановской без задней мысли.
Аноним 13/02/20 Чтв 18:58:38 548805227
>>548798
> При помощи каких средств человек, будучи в ракете/шаттле/пепелаце, может без особого вреда пересечь пояс радиации? Насколько мне известно, солнечная радиация - это пиздец какая уберхуйня. И каким образом, в таком случае, можно осуществить полёты на Марс?
Вон я выше: >>548584 принцип активного гамма-щита описал. Там, конечно, тоже свой геморрой, но всё воплотимо и законам физики не противоречит.
Аноним 13/02/20 Чтв 20:23:58 548812228
>>548805

Ну такое себе, кондёр пробьет быстрее чем ты успеешь отреагировать и снизить напругу.
Аноним 13/02/20 Чтв 23:26:56 548825229
>>548742
А было бы неплохо
14/02/20 Птн 00:47:59 548841230
image.png (134Кб, 320x240)
320x240
Знаю что не про космос, но на дваче нет больше умных людей
Чем так опасен графит из ядерного реактора? У C11 период полураспада 11 минут, остается C14. А у него только бета-минус распад, от которого несложно защититься. Почему при ликвидации аварии на ЧАЭС всего по паре минут люди работали на крыше?
Аноним 14/02/20 Птн 01:11:50 548846231
>>548841
Он ГОРИТ. И оказывается в воздухе, его пыль попадает повсюду. Внутри организма от такого распада не защитится, будет пиздец.
Аноним 14/02/20 Птн 12:48:20 548897232
Разве пузырь не сдулся? Когда последний раз читал о нем, то там было что то вроде "эксперименты не хотят подтверждать такую возможность"
Аноним 14/02/20 Птн 15:33:17 548907233
>>548568
>кориолисовы силы
не понимаю как они работают, объясни пожалуйста
Аноним 14/02/20 Птн 15:50:44 548908234
>>548907
Ну это типа виртуальная сила, возникающая при работе с вращающимися системами отсчёта.
14/02/20 Птн 15:53:07 548909235
Сколько БЭР получит космонавт при перелёте с Земли к Марсу? Говорят при спокойном солнце до третей стадии лучевой болезни, правда?
14/02/20 Птн 16:27:55 548919236
>>548846
Почему тогда на крышах не работали в КИПах но дольше?
Аноним 14/02/20 Птн 16:28:16 548920237
>>548919
сега приклеилась
Аноним 14/02/20 Птн 17:34:18 548932238
sergiu-ikarus-f[...].jpg (350Кб, 1065x1600)
1065x1600
>>548897
Пузырь - это светлый идеал, идея, к которой мы стремимся, и пофиг, что эксперименты не подтверждают, мы придумаем новые и будем шатать пространство и время, пока не создадим пузырь, каскадный резонанс или портал в Эквестрию!
Аноним 14/02/20 Птн 17:43:47 548933239
>>548897
Разве кто-то делал эксперименты по пузырю кроме того предвзятого фаната стартрека?
Аноним 14/02/20 Птн 17:49:19 548936240
Аноним 14/02/20 Птн 17:54:40 548937241
>>548936
Сонни Уайт это и есть тот самый фанат стартрека
14/02/20 Птн 17:57:08 548938242
>>548618
метеориты не радиоактивны
Аноним 14/02/20 Птн 18:03:08 548940243
>>548937
Откуда ж мне это знать было?
Аноним 14/02/20 Птн 18:09:32 548942244
Аноним 14/02/20 Птн 19:03:46 548947245
>>548841
Тем, что он загрязнен ВСЕМ ЧЕМ ТОЛЬКО МОЖНО, ибо его пернуло из раскочегаренного реактора, какие нахуй С11-14? Там блять В-С-Е и альфой и беткой и гаммой ебашило что мама не горюй
Аноним 14/02/20 Птн 19:06:53 548948246
>>548689
>Сурдин.
Борис Штер
Сергей Попов
Кирилл Маслеников

Из космонавтов никого больше не знаю. На постнауке можно посмотреть каких-то малоизвестных.
Аноним 14/02/20 Птн 19:09:56 548949247
>>548947
Насколько я слышал. Самая ёба это нейтронка. Если альфа бета хуйня, гамма опасна, то вот нейтронка это лютая зараза.
И измеряются они по разному.
Аноним 14/02/20 Птн 20:25:49 548954248
А вот говорят, что вот всякие космотелескопы (которые не работают в инфракрасной области спектра) сгорят если повернутся к солнцу. А почему нельзя натянуть тёмный фильтр? Меркурий бы посмотрели, солнце там, всякие астероиды что засвечиваются.
Аноним 14/02/20 Птн 20:55:06 548959249
>>548954
Космотелескопы это дорогая штука. На него еще можно дилдо повесить, мигалку, гудок и гирлянду. Можно, но зачем? У него есть главная задача, а точнее список из задач, которых придерживаются при проектировке. Если там нет необходимости пырить на меркурий и внутренние астероиды, то никто не будет лишнюю хуйню городить. Там к тому же не много астероидов внутри орбиты Земли, их почти всех повыкидывало в орбиты которые имеют апогей выше орбиты Земли, и их так можно обнаружить смотря наружу от Солнца. Меркурий лучше изучать АМС, никакой телескоп на орбите Земли не даст данных лучше орбитера.
Аноним 14/02/20 Птн 21:00:40 548960250
>>548954
>(которые не работают в инфракрасной области спектра)
Те которые работают в инфракрасном сгорят еще быстрее, лол. Их же охлаждать специально надо, и в лагранж2 вешают, чтобы они не видели света Солнца.
Аноним 14/02/20 Птн 21:44:46 548969251
>>548959
>Меркурий лучше изучать АМС, никакой телескоп на орбите Земли не даст данных лучше орбитера.
Ты только что любую планету.
Аноним 15/02/20 Суб 00:10:47 548989252

>>547978
Ну вторая 100% не врёт, ну тупые, она же читает просто!
Аноним 15/02/20 Суб 02:19:22 548996253
43243214.png (4Кб, 800x600)
800x600
Почему все планеты крутятся вокруг солнца по фиолетовой орбите, и нет таких планет, которые бы крутились как-бы "сверху-вниз", по синей орбите? Почему почти все крупное говно обращается по плоскости фиолетовой орбиты?
Аноним 15/02/20 Суб 02:46:52 548997254
>>548996
Потому что планеты образовались вместе с Солнцем из одного и того же облака межзвездного газа. Просто прикинь, как могло бы вращаться такое облако, и увидишь, что нет каких-то его "частей" двигающихся по фиолетовой орбите.

Но вообще, конечно, у всех планет есть какое-то, пусть небольшое, отклонение от плоскости эклиптики. У Плутона оно аж 17 градусов.
Аноним 15/02/20 Суб 02:47:28 548998255
>>548997
>нет каких-то его "частей" двигающихся по фиолетовой орбите
По синей то есть. Ну ты понел.
Аноним 15/02/20 Суб 04:03:04 549000256
Стикер (255Кб, 500x500)
500x500
>>548997
>у всех планет
А Плутон здесь при чём?
Аноним 15/02/20 Суб 07:57:07 549008257
>>548205
Капоне, залогинься.
Аноним 15/02/20 Суб 08:00:36 549009258
>>549008
>Гудзон
>Капоне
Да там всего два фрага было. Предположительно. Вы ничего не докажете.
Аноним 15/02/20 Суб 09:11:01 549010259
Аноним 15/02/20 Суб 09:11:07 549011260
>>548998
>По синей орбите
Запишу это, пожалуй
Аноним 15/02/20 Суб 10:16:50 549014261
>>549011
Третьего дня по синей орбите спутал спутники, в итоге оказался на Ио, а там, как известно, 95-й антиматерии нет, только 92 и кварк-глюонная плазма. Реакторщик на следующем ТО обнаружит следы Z-бозонов в аннигиляционной камере и опять будет хуесосить "сэкономил 200 кредитов на одной заправке - платишь 20000 за новый инжектор", блжад.
Спейсач, посоветуй, как с камеры оттереть следы аннигиляции гелий-антигелий, у меня ж гарантия на реактор пиздой накроется.
Аноним 15/02/20 Суб 11:06:35 549018262
>>549014
Слишком много профессионального жаргона. Аннигиляция это не "прилипло", а "поцарапало", так что никак.
Аноним 15/02/20 Суб 13:16:18 549030263
>>548997
Но почему-то дохуя наклоненных экзопланет находят. Причем это какие-то юпитеры которые и под 45 градусов, и под 90, и под 230 крутятся относительно орбит других планет
Аноним 15/02/20 Суб 13:24:44 549031264
>>549030
Скорее всего пойманное межзвездное говно. Собственный протопланетный диск в конечном итоге скукоживается плюс-минус в единую плоскость эклиптики.
Аноним 15/02/20 Суб 13:49:35 549033265
Аноним 15/02/20 Суб 17:26:42 549052266
EN5nRQtU4AAkn3u.jpg (774Кб, 1100x1539)
1100x1539
>>549030
В раннее время образования планетной системы может происходить хаос, когда планеты еще не заняли стабильные позиции, большие планетоземали и планеты могут близко друг к другу подходить, сталкиваться и выкидывать друг друга на орбиты с высокой эклиптикой и наклонениями. Думаешь почему Венера вращается в другую сторону? Как Луна образовалась? Кто въебал по Марсу так что он потерял магнитное поле? Кто наклонил Уран? В других планетных системах может быть еще хуже, там могут быть объекты тяжелее Юпитера, и не один, а несколько. Они таких дел могут натворить, пиздец всему. Могут друг друга ускорить на высокие эксцентриситеты и отправить на хуёвые наклонения по резонансу Лидова-Козаи.
Аноним 15/02/20 Суб 17:33:55 549054267
images.jpeg (6Кб, 225x225)
225x225
>>549052
>Кто въебал по Марсу так что он потерял магнитное поле? Кто наклонил Уран?
Аноним 15/02/20 Суб 17:48:07 549055268
Есть ли какие-то ограничения, соотношения, закономерности у массы планеты и звезды?
Может ли быть так, что образовалась осне мелкая звезда и дохуища планет почти коричневых карланов?
Или какое-то правило есть что минимум 90% массы газопыли обязательно звездой становится, например?
И обратно - если гигантская звезда, значит много материала, значит ли что у нее был жирный протопланетный и она дохуя планет наделала?
Аноним 15/02/20 Суб 22:37:55 549096269
Какие компьютеры используют космоастронавты?
Аноним 15/02/20 Суб 22:39:29 549097270
>>549096
Обычные Thinkpad'ы.
Некоторые обычные вещи им приходится перепокупать через левые фирмы которые просто чутка перехуячивают их и благодаря этому можно пройти space grade сертификацию.
Аноним 15/02/20 Суб 23:22:04 549110271
>>549097
>Обычные Thinkpad'ы.
От леново? Какая у них начинка?
Аноним 15/02/20 Суб 23:24:27 549113272
Можно ли жить внутри Энцелада в подводном океане? Там же считай вода может быть как на Земле, просто море. Там можно и теплицы ставить, и водоросли выращивать, и рыбу завезти. Правда эту рыбу потом будет в космос джетами выбрасывать, но что поделать, будет космический рыбный мусор.
Аноним 15/02/20 Суб 23:26:08 549114273
>>549113
>Можно ли жить внутри Энцелада в подводном океане?
Да

мимо рачок с Энцелада
Аноним 16/02/20 Вск 00:27:10 549131274
>>549055
При зажигании звезды пыль сдувается и с увеличением массы звезды увеличивается и мощность ветра сдувающего. Соответственно чем больше звезда тем меньше суммарная масса компаньонов. Если конечно это не кратная звезда.
А ещё когда облако коллапсирует, оно в основном падает в центр массы, и лишь малой их части получается закрутится в эклиптичную орбиту. Иначе требуются какие-то специфичные условия. Всякое бывает даже 10 кратные звёзды. И какой-нибудь желтуш с кучей коричнивых карланов.(правда карланы в сумме всё равно будут меньше желтуша)
Аноним 16/02/20 Вск 00:28:58 549132275
>>549131
>Всякое бывает даже 10 кратные звёзды
Это кто? Самое больше же септернарные. АР кассиопеи и Ню скорпиона
Аноним 16/02/20 Вск 00:42:25 549135276
>>549132
Ну такое не обнаружили. Но я в спейс энжене видел. Но не исключено же.
Аноним 16/02/20 Вск 00:44:43 549136277
>>549135
В целом не исключено, но и не подтверждено.
А ну как есть какие фундаментальные ограничения из-за которых просто невозможны стабильные долговременные конфигурации 9+ объектов звездных масс? Ну как спутник спутника, например.
Аноним 16/02/20 Вск 01:18:10 549144278
>>549136
Формально 100 000 кратная система существует и наблюдается(шаровое скопление). А 9+ объекты не самое сложное представить. Если знать что все тесные кратные звёзды обычно делятся по парам.(Тот же Мицар).
Да и не стоит забывать об дискретности границы звезда-планета. Набери десяток коричневых карликов едва достигающие минимальной массы, вот тебе и звёздная система.
Аноним 16/02/20 Вск 01:24:17 549145279
>>549144
А карланы за звезды считаются-то?
Аноним 16/02/20 Вск 15:20:52 549180280
>>549145
В том и вопрос. Одни говорят да, другие нет, третьи говорят что-то среднее. Как по мне что планета, что звезда попадают под принцип Эскобара. В принципе лишь проблема названия. Но в принципе там термоядерные реакции идут, хоть и не долго.
Аноним 16/02/20 Вск 16:42:58 549193281
анончик, поясни дебилу-гуманитарию
если в работающем водородном ракетном движке встретятся два шальных атома дейтерия и сольются в синтезе
общая температура струи в этот момент возрастет на миллипиздрический градус или упадет?
Аноним 16/02/20 Вск 16:55:47 549195282
>>549193
Возрастет, потому что результаты слияния будут разлетаться как в жопу ужаленные. Но вообще-то это гипотетический вопрос, потому что скорости атомов в струе движка слишком маленькие, чтобы синтез вообще произошел.
Аноним 16/02/20 Вск 17:06:28 549196283
>>549195
а если детонационник?
Аноним 16/02/20 Вск 18:13:37 549202284
>>549193
>>549196
Там порядки не те вообще.
Гипотетически действительно может два шалных атома в экстазе слиться и выдать сколько-то мегаэлектронвольт, прямо как миллион молекул которые сгорели. Миллион. Что на двадцать порядков меньше, чем химически реагирует молекул за секунду в двигателе.
Двадцать порядков, парень.
Сказать, что твоя ядерная реакция будет незаметна - ничего не сказать.
Аноним 16/02/20 Вск 18:40:20 549204285
>>549202
а нам шо, уже таки нужен термоядерный взрыв?
надо же просто чтобы десятки кило топлива в секунду горели ЯРЧЕ
Аноним 16/02/20 Вск 18:41:20 549205286
Аноним 16/02/20 Вск 18:54:36 549206287
>>549205
как соскочить с ограничений химки
Аноним 16/02/20 Вск 18:55:51 549207288
>>549206
Ядерные реакции-то тут при чем? Ты либо реактор пилишь, либо химию жжешь, гибрида быть не может.
Аноним 16/02/20 Вск 19:07:42 549208289
>>549207
идея в том что бы запустить слабый вялотекущий синтез на химическом зажигании
Это тред тупых вопросов! хуле ты хотел?
Аноним 16/02/20 Вск 19:10:44 549209290
>>549208
Невозможно в принципе. Порядки энергий не те.
Десятки-сотни электронвольт от химии против миллионов у ядерных реакций. Ты не разгонишь химией атомы чтобы реакции были. Никак. В принципе.
Аноним 16/02/20 Вск 20:22:12 549217291
>>549207
>Ядерные реакции-то тут при чем? Ты либо реактор пилишь, либо химию жжешь, гибрида быть не может.
Может.

Во-первых, простую гибридную схему я тут неоднократно описывал. Берём ядерный генератор на вторичных электронах (кпд ~20%), помещаем в криогенный бак с химическим топливом и раскочегариваем на максимум. Топливо кипит (попутно охлаждая реактор), прёт под давлением в камеру сгорания и сгорает традиционным способом. При этом электропроводность раскалённых газов повышается. На сопло надет запитанный от реактора индукционник, доразгоняющий газы до неприличных скоростей.

Во-вторых, есть сведения о том, что у полностью ионизированных атомов (т.е. когда внутренние электронные оболочки тоже обрывают) начинает скакать период полураспада. Причём даже у стабильных изотопов. А вот это уже прямая дорожка к пока ещё малоизученным переходным реакциям. Алсо, это не единственный способ дестабилизации стабильного ядра. Есть ещё тема с ядерными изомерами и избыточным вращением ядер.
Аноним 16/02/20 Вск 20:27:04 549218292
>>549217
У тебя топливный бак работает минуты три, после чего он пустой. Не дороговато ли йоба-генератор туда пихать?
Сведения про полураспад скинь, хочу работы глянуть.

Мне все эти методы очень сомнительны, если металлический водород метастабилен - вот где возможность на порядок поднять КПД движка без особого его усложнения.
Аноним 16/02/20 Вск 20:50:29 549222293
>>547634 (OP)
Почему сотрудники фсбкгб вместо того, чтобы заниматься наукой и физкультурой — бухают и наркотики едят, похотью и эротизмои занимаются?

Нас же так завоюют соединенные США штаты америки.
Аноним 16/02/20 Вск 20:55:47 549224294
>>549218
>У тебя топливный бак работает минуты три, после чего он пустой. Не дороговато ли йоба-генератор туда пихать?
Он работает существенно дольше, потому что камера сгорания и расход очень маленькие. Вклад химии в тягу здесь небольшой, главная задача химии - нагреть пропеллент, избавив нас от ебли с ионизацией и от необходимости лить в баки дорогущий ксенон. Второстепенная задача - охлаждение реактора, используя паразитное тепло для повышения давления в цикле фазового перехода. Это тоже небольшая добавка в тягу, но она есть. А основную тягу производит именно электромагнитная часть, доразгоняющая струю. Это такой VASIMR курильщика, у которого составляющие части нейтрализуют дефекты друг друга.

>Сведения про полураспад скинь, хочу работы глянуть.
https://www.epj-conferences.org/articles/epjconf/pdf/2015/12/epjconf_cgs2015_05003.pdf
Алсо, здесь https://habr.com/ru/post/438750/ по ссылкам пошарься, тоже много интересного.
Там же упоминания про кубическую гош-модификацию азота. Это ещё не металлический водород, но вполне себе представитель супербарической химии (когда энергии сжатия сопоставимы с энергиями электронных оболочек), метастабильный при нормальных условиях.
Аноним 16/02/20 Вск 20:58:59 549225295
>>549222
они нас давно завоевали
где при Петре 1
Аноним 16/02/20 Вск 23:27:19 549246296
-67185996323608[...].jpg (54Кб, 474x501)
474x501
>>549225
>Пётр I, дата смерти: 8 февраля 1725 г.
>США, дата образования: 4 июля 1776 г.
Сраный ты дебил.
Аноним 16/02/20 Вск 23:38:39 549248297
>>549218
>если металлический водород метастабилен
если он стабилен после получения в адском давлении то при переходе в газ он и ебануть может, нет?
Аноним 16/02/20 Вск 23:40:36 549249298
>>549246
>не знать о Священной Британской Империи глобального капитала, которая решает давать ли топовое лекарство очередному русскому царьку или он обойдется из-за плохого поведения.
Ты охуел холоп не знать о Хозяевах своих хозяев?
Аноним 16/02/20 Вск 23:42:36 549250299
>>549246
>США, дата образования
это когда пацаны из Ост Индской компании решили сделать себе собственное государство без феодалов и даже с флагом решили не заморачиваться?
Как то ты плохо понимаешь что такое США
Аноним 16/02/20 Вск 23:56:57 549252300
>>549248
Так и надо чтобы ебанул. Только контроллируемо.
Аноним 16/02/20 Вск 23:57:28 549253301
>>549250
Ну хуй с сосача конечно лучше самих жителей США знает, когда образовались США, но своё без сомнения интересное толкование истории можешь оставить при себе.
Аноним 16/02/20 Вск 23:58:00 549254302
>>549224
>Он работает существенно дольше, потому что камера сгорания и расход очень маленькие. Вклад химии в тягу здесь небольшой, главная задача химии - нагреть пропеллент, избавив нас от ебли с ионизацией и от необходимости лить в баки дорогущий ксенон. Второстепенная задача - охлаждение реактора, используя паразитное тепло для повышения давления в цикле фазового перехода. Это тоже небольшая добавка в тягу, но она есть. А основную тягу производит именно электромагнитная часть, доразгоняющая струю. Это такой VASIMR курильщика, у которого составляющие части нейтрализуют дефекты друг друга.
Чет мне не кажется, что тут математика сходится.
Давай-ка рассмотрим твои диванные выкладки, может чего упускаешь.
Аноним 17/02/20 Пнд 00:22:58 549256303
>>549252
меня больше прикалывает что при огромной энергии солнца в космосе, водородное топливо можно хранить в сгоревшем виде в виде льд, что даже позволит сохранить все топливо при ударе микрометеоритов,
с другими топливами такой фокус не пройдет. с металлическим водородом то же
надо просто допетрить как из 50 кг/c топлива заставить просинтезироваться хотя бы миллиграмм дейтерия/
при таких раскладах таки можно будет делать в космосе некий гешефт
Аноним 17/02/20 Пнд 00:26:37 549257304
>>549256
> водородное топливо можно хранить в сгоревшем виде в виде льд
Парень, ты...
Какой смысл это делать, если оно на низшем энергетическом уровне? Это не топливо теперь.

>с металлическим водородом то же
Ты либо недописал, либо знаешь значение.

>надо просто допетрить как из 50 кг/c топлива заставить просинтезироваться хотя бы миллиграмм дейтерия/
>при таких раскладах таки можно будет делать в космосе некий гешефт
Если у тебя есть термоядерный реактор, работающий, мобильный, умеющий, то доработать до чистого водорода вопрос техники и доводки уже. Или бор тащи, нахуя водород?
Аноним 17/02/20 Пнд 00:26:46 549258305
>>549253
так жители США сами в курсе, на форчах дохуя бугурта по этому поводу
уж всяко не дремучую пидораху об этом спрашивать. почему Российская империя и СССР раз за разом сосали исправно британский хуй, а США полностью проигнорировали наглую передачу британцами ядерных и авиационных секретов Совку
Аноним 17/02/20 Пнд 00:29:31 549259306
>>549257
>Какой смысл это делать, если оно на низшем энергетическом уровне? Это не топливо теперь.
>
электролиз водорода слышал? ты еще спроси откуда в космосе взять энергию на это

>Если у тебя есть термоядерный реактор
нахуя мне реактор если речь идет о двигателе? разницу между ними знаешь?
Аноним 17/02/20 Пнд 00:31:11 549260307
>>549259
>электролиз водорода слышал? ты еще спроси откуда в космосе взять энергию на это
Вот именно, ты везешь отработку чтобы потом с помощью энергии разделить ее на топливо чтобы получить энергию.
Наркоман что ли? У тебя уже есть энергия в космосе, нахуя ты отработку, выхлоп тащишь?

>нахуя мне реактор если речь идет о двигателе? разницу между ними знаешь?
Да, знаю. У тебя говеный двигатель.
Аноним 17/02/20 Пнд 00:55:33 549261308
>>549260
>>549260
>от именно, ты везешь отработку чтобы потом с помощью энергии разделить ее на топливо чтобы получить энергию.
>Наркоман что ли? У тебя уже есть энергия в космосе, нахуя ты отработку, выхлоп тащишь?
расскажи как ты собираешься хранить в космосе топливо
где в любую секунду тебе могут бак кхуям продырявить навылет?
ах да еще же солнце 250 градусов жара дает
а у тебя все топливо должно быть температуры не -1 градус на ГОРАЗДО ниже
холод без потери массы в космосе гораздо дороже электроэнергии

>У тебя говеный двигатель.
я знаю, но почему ты так бомбишь?
Аноним 17/02/20 Пнд 00:59:19 549262309
>>549261
>расскажи как ты собираешься хранить в космосе топливо
Так же как и не в космосе. В бочках.
>где в любую секунду тебе могут бак кхуям продырявить навылет?
А могут и не продырявить. И статистика показывает, что обычно нихуя не дырявит.
>ах да еще же солнце 250 градусов жара дает
А в тени 4 градуса.
>а у тебя все топливо должно быть температуры не -1 градус на ГОРАЗДО ниже
У тебя какая-то афазия пошла. Беру гидразин нахуй.
>холод без потери массы в космосе гораздо дороже электроэнергии
Поэтому не холожу.

>я знаю, но почему ты так бомбишь?
Потому, что я не понимаю, чего ты пытаешься добиться, у тебя идеи уровня тех, которые придумываются на пике Даннинга-Крюгера.
Аноним 17/02/20 Пнд 01:10:03 549264310
>>549262
>А в тени 4 градуса.
>
в тени ЧЕГО
тень в космосе РЕДКОСТЬ
Аноним 17/02/20 Пнд 01:11:31 549265311
>>549262
>И статистика показывает
статистика говорит что если ОДИН крохотный по меркам космоса объект за ничтожный срок дырявило несколько раз
то при нормальной эксплуатации космоса дырявить будет постоянно
Аноним 17/02/20 Пнд 01:12:08 549266312
>>549262
>Поэтому не холожу.
>
кислород в виде газа будешь хранить?
Аноним 17/02/20 Пнд 04:24:18 549273313
>>549262
>Так же как и не в космосе. В бочках.
Нахер бочки-то? Намешать ферритового порошка да удерживать магнитом, пусть хоть запродырявливаются.
Аноним 17/02/20 Пнд 08:07:20 549279314
>>549264
>в тени ЧЕГО
>тень в космосе РЕДКОСТЬ
В тени САМОГО СЕБЯ.

>>549265
>статистика говорит что если ОДИН крохотный по меркам космоса объект за ничтожный срок дырявило несколько раз
>то при нормальной эксплуатации космоса дырявить будет постоянно
На НОО, где говно постоянно летает. И то без катастроф, т.к. достаточно дамажащие вещи отслеживаются, а мелочь отлавливается микрометеоритной защитой.

>>549266
>кислород в виде газа будешь хранить?
В виде азотного соединения, ну, скажем, тетраоксид азота.

>>549273
>Нахер бочки-то? Намешать ферритового порошка да удерживать магнитом, пусть хоть запродырявливаются.
А нахера решать непроблемы вообще? Тем более для дальних человеческих полетов до сих пор носителя нет, так что летать будут роботы, а на их безопасность можно положить болт обратно пропорциональный их стоимости, но все же болт.
Аноним 17/02/20 Пнд 09:36:05 549283315
>>549279
>В тени САМОГО СЕБЯ.
если бы Ньютон это позволял давно бы летали корабли на ядерных движках
>а мелочь отлавливается микрометеоритной защитой.
МКС расскажи про защиту, вечно дырявая летает
Аноним 17/02/20 Пнд 09:38:34 549284316
>>549283
>если бы Ньютон это позволял давно бы летали корабли на ядерных движках
Ты че? ТЫ ЧЕ!? У тебя сторона которая в сторону от солнца смотрит тоже им освещена что ли, наркоман!?
>МКС расскажи про защиту, вечно дырявая летает
Защита дырявая. Не МКС. Все как и положено, нехуй им рассказывать, побольше твоего знают.
Аноним 17/02/20 Пнд 09:44:10 549285317
>>549284
>Ты че? ТЫ ЧЕ!? У тебя сторона которая в сторону от солнца смотрит тоже им освещена что ли, наркоман!?
>
ааа так ты не в курсе о перегреве в космосе?
Господа, кто его сюда пустил?
Аноним 17/02/20 Пнд 09:46:32 549286318
>>549285
Блядь, ты тупарь, радиаторы в тени расправляй, еблан сраный, это ты как сюда попал, дегенерат?
Аноним 17/02/20 Пнд 09:54:06 549287319
>>549286
ты видал какие радиаторы на МКС?
а им суперхолод не надо поддерживать
у них радиаторы только от солнца и крох электричества
а тебе здоровенные криогенные баки надо в порядке содержать

хотя ты рвешься как подзаборное быдло, не мудрено что ты ничего не понимаешь
Аноним 17/02/20 Пнд 10:30:58 549288320
>>549287
>а тебе здоровенные криогенные баки надо в порядке содержать
Какие. Сука. Криогенные. Баки.
Долбоеб, блядь, читай тред, а не пукай первое что в голову взбредет.
Аноним 17/02/20 Пнд 11:04:47 549291321
>>549113
свет откуда водоросли будут брать?
Аноним 17/02/20 Пнд 11:07:13 549292322
>>549291
Свет не нужен, светобляди не люди можно на химии жить.
Аноним 17/02/20 Пнд 11:24:00 549294323
>>549288
то есть ты поехавший влез в чужой разговор и начал бомбить?
речь шла о паре водород+кислород
да ты просто ебнутый наглухо
Аноним 17/02/20 Пнд 11:27:46 549295324
>>549294
Какой паре, еблан, речь шла про нихуя ненужный электролиз воды.
Съеби нахуй в зогач, общайся с равными тебе по интеллекту, чмо.
Аноним 17/02/20 Пнд 12:29:35 549298325
На дрэгоне и прогрессе объём герметичный же, теоретически возможно в них человеку в скафандре с запасом кислорода до мкс долететь?
Аноним 17/02/20 Пнд 13:16:15 549302326
>>549298
Обчитался Майн Рида и зайцем решил?
Аноним 17/02/20 Пнд 14:12:34 549310327
>>549302
Нет, просто интересна чисто гипотетическая возможность.
Вдруг на грузовых кораблях какие-либо закритические перегрузки и человеку просто не пережить?
Аноним 17/02/20 Пнд 14:14:23 549311328
>>549310
Нет, там же и эксперименты всякие перевозят. Перегрузки не сильно от пилотируемых отличаются.
Аноним 17/02/20 Пнд 15:33:09 549334329
Вопрос на счёт эволюции: получается что животные, которые быстрее рождаются / умирают и дают много потомства в будущем будут более приспособлены к жизни?
А если взять людей, допустим есть семья которая заводит детей в 20 лет, а другая в 30 - через несколько поколений у первой семьи выйдет на 1 поколение больше чем у второй, и в целом это поколение будет более устойчиво к внешних раздражителям и более развито или каК?
Аноним 17/02/20 Пнд 15:38:54 549335330
>>549334
>на счёт
Значение знаешь?
>эволюции
>/spc/
Звездной, надеюсь.
>получается что животные
Блядь.
>получается что животные, которые быстрее рождаются / умирают и дают много потомства в будущем будут более приспособлены к жизни?
Вид быстрее приспособится к окружению.
>А если взять людей, допустим есть семья которая заводит детей в 20 лет, а другая в 30 - через несколько поколений у первой семьи выйдет на 1 поколение больше чем у второй, и в целом это поколение будет более устойчиво к внешних раздражителям и более развито или каК?
Никак. Через 3 поколения даже селекцией не добиться больших генетических различий у людей.
У мушек проходят десятки поколений перед тем, как эволюционно обоснованные изменения проявляются достаточно явно. И это в крайне тяжелых условиях.
Ну то есть если ты возьмешь поселок скандинавов и поселишь их в африке, обеспечив едой и высокой смертностью, то через лет 300-500 их потомки будут чернокожими. Это всего лишь мысленный эксперимент с кучей допущений. Нет, негры не побелеют, отбора нет, который бы позволял больше выживать более светлокожим.

А вообще это не относится к космосу, чому в /sci/ не спрашивал?
Аноним 17/02/20 Пнд 15:40:43 549337331
>>549335
Ну тут тред тупых вопросов, а где больше спросить я и не знаю. Ну чтобы нормально отвечали, а не бред.
Аноним 17/02/20 Пнд 15:44:00 549344332
>>549337
Если по-хорошему, то надо в /sci/ спрашивать такое.
Но да, на всей борде только тут могут пояснить, почему пельмешки всплывают при варке и почему комариный укус поначалу не чувствуется.
Аноним 17/02/20 Пнд 15:48:02 549349333
ХЗ кому из вас отвечать, но радиаторы нинужны, капельные холодильники излучатели придуманы давно.
Аноним 17/02/20 Пнд 15:50:23 549352334
>>549349
Где посмотреть на действующий аппарат с таким на борту?
Аноним 17/02/20 Пнд 15:50:24 549353335
>>549349
И не реализованы ни разу до сих пор.
Когда реализуют - будем всерьез рассматривать.
Только это... Они тоже радиатор.
Аноним 17/02/20 Пнд 16:19:02 549360336

не хочу никого расстраивать, но с капельными холодильниками обосрались
тоже плохо в школе учились, как КРУПНОБУКВА
Аноним 17/02/20 Пнд 16:21:30 549361337
>>549360
Твоё мнение по любому вопросу здесь уже никого не интересует, обтекай.
Аноним 17/02/20 Пнд 16:22:33 549362338
>>549361
а я разве с тобой, петух, разговаривал?
ты же неуч, с двойкой по физике.
хуже гуманитария
Аноним 17/02/20 Пнд 16:23:06 549363339
Аноним 17/02/20 Пнд 16:26:38 549364340
>>549362
ИК-излучение от твоей сраки перегревает тред.
Аноним 17/02/20 Пнд 16:34:34 549367341
>>549363
Наземные-то да. А на бумаге еще лучше.
Осталось только в космосе не расплескать.
Вот пусть аппарат слетает и успешно не сгорит - и будем рассматривать капельки, никто не против.

>>549364
Ну зачем ты с петушней пререкаешься? Шли его на хуй и все.
Аноним 17/02/20 Пнд 16:35:14 549370342
1
Аноним 17/02/20 Пнд 16:39:34 549373343
>>549363
>>549367
тоже как ты были с физикой не в ладах
а потом в космосе вдруг выясняется что капли переизлучают не только в космос но и друг на друга
что не играет роли если выкинуть эти атомы
но очень играют роль когда ты их возвращаешь.
двоечники блять

главное что нам не надо в космосе топливо хранить да и вообще нам космос ваш нинужон!
Аноним 17/02/20 Пнд 20:38:18 549472344
Почему в чд теряется куча всякой информации, но информация о моменте вращения и заряде в этот список не входит и сохраняется?
Аноним 17/02/20 Пнд 21:14:18 549483345
>>549472
Тому що законы физики CPT-симметричны, никакие взаимодействия с зарядом и моментом вращения не могут создавать их из ниоткуда и тратить в никуда.
Аноним 17/02/20 Пнд 21:22:48 549488346
>>549483
Но ведь другая-то информация теряется, хотя это тоже нарушает дохуя законов и делает физикам бугурт. Почему те нарушают, а эти нет?
Аноним 17/02/20 Пнд 21:35:11 549492347
1558267306831.jpg (1604Кб, 3042x2569)
3042x2569
Допустим у нас есть реактор производящий антиматерию - антиводород. И мы находясь в межзвёздном пространстве начинаем его генерировать и выпускать в открытый космос. Этот антиводород сможет там накапливаться или же он будет аннигилировать с пылью? Можно ли создать звезду из антиматерии если генерировать газ очень долго, или же наличие пыли в пространстве галактики будет постоянно убивать любую генерацию антивещества?
Аноним 17/02/20 Пнд 21:36:22 549493348
>>549488
а что именно за информация теряется и что такое информация, ты знаешь? Определение информации в студию. Буггурт теоретических физиков чисто теоретический, кому не похуй вообще.
Аноним 17/02/20 Пнд 22:15:27 549512349
15145839437381.jpg (18Кб, 340x309)
340x309
Допустим у нас есть антиматерия типа антиплутоний на уровне почти критической массы - будет ли взрыв мощнее если мы не доводя до критической массы прореагируем антиплутоний с материей, или же взрыв получится мощнее, если мы доведем антиплутоний до критического состояния и инициируем цепную реакцию приводящую к антиядерному взрыву?
Аноним 17/02/20 Пнд 22:18:32 549514350
>>549512
Однохуйственно. Е равно эм цэ квадрат. Эм в обоих случаях одинаковое.
Аноним 17/02/20 Пнд 22:20:37 549517351
>>549492
Но ведь можно искусственно очистить пространство от пыли и создавать антиматерию в чистом вакууме, а потом запускать по планетам злобных инопланетиан, вот было бы здорово.
Аноним 17/02/20 Пнд 22:21:03 549518352
>>549514
(уточнение: в том случае, если продукты антиядерного деления тоже анигилируют об материю. Если нет, то анигиляция в первом случае, естественно, мощнее.)
Аноним 17/02/20 Пнд 22:21:55 549520353
Я слышал, что антиматерия это материя двигающиеся назад во времени. Будет ли взрыв антиплутона инверсирован во времени?
Аноним 17/02/20 Пнд 22:23:51 549522354
Поясните за ядерный буксир от роскосых. Хуита али нет?
Аноним 17/02/20 Пнд 22:24:35 549523355
>>549514
При ядерном взрыве в энергию переходит только небольшая доля массы, а при аннигиляции - вся (правда, часть будет в виде нейтрино, которых считай что нет, но и остального за глаза хватит).
https://ru.wikipedia.org/wiki/%D0%90%D0%BD%D0%BD%D0%B8%D0%B3%D0%B8%D0%BB%D1%8F%D1%86%D0%B8%D1%8F#%D0%A1%D1%80%D0%B0%D0%B2%D0%BD%D0%B5%D0%BD%D0%B8%D0%B5_%D0%B2%D1%8B%D0%B4%D0%B5%D0%BB%D0%B5%D0%BD%D0%B8%D1%8F_%D1%8D%D0%BD%D0%B5%D1%80%D0%B3%D0%B8%D0%B9
Аноним 17/02/20 Пнд 22:27:21 549524356
>>549522
Можешь смело ставить анус, что его не будет в работоспособном виде в космосе при твоей жизни.
Аноним 17/02/20 Пнд 22:30:41 549526357
1558295644590.jpg (370Кб, 957x952)
957x952
>>549514
Нет же - в случае аннигиляции будет одна реакция аннигиляции внутрь шара антиплутона, а в случае антиядерного взрыва будет сначала распад антиплутона с распадом на антиплутоновые изотопы внутри сферы антиматерии, которая будет создавать фронт из продуктов распада распространяющийся во всех направлениях и уже так будет аннигилировать с материей и в зависимости от количества и концентрации материи вокруг сферы антиядерного взрыва будет разное выделение энергии от аннигиляции продуктов распада. То есть взрыв антиядерной бомбы на орбите Земли может например создать кольцо из антиматерии, которое будет аннигилировать всё с чем будет вступать в контакт. Также можно например взорвать антиядерную бомбу на траектории полёта космического корабля или астероида и он войдёт в сферу из античастиц, которые начнут аннигилировать с поверхностью корабля или астероида. Это совсем другое выделение энергии кароче
Аноним 17/02/20 Пнд 22:52:45 549535358
>>549522
>Поясните за ядерный буксир от роскосых. Хуита али нет?
Идея здравая, воплощение с таким руководством - сам понимаешь.

>>549520
>Я слышал, что антиматерия это материя двигающиеся назад во времени. Будет ли взрыв антиплутона инверсирован во времени?
Мы пока мало что знаем об антиматерии и о недокументированных особенностях симметрии. Неизвестно даже, как она гравитационно с материей взаимодействует, что у неё с макроскопической термодинамикой и.т.п. Пока только сугубо теоретически сходятся CPT-уравнения, а на практике может вылезти какая угодно залупа.
Аноним 17/02/20 Пнд 23:10:09 549540359
motor drive.jpg (64Кб, 349x267)
349x267
>>549522
Это сложная тема так как требуется создать многие составляющие компоненты ядерного буксира. Если просто создать ракетный двигатель сложно, то представь как сложно создать ракетный двигатель в котором еще ядерные элементы как-то связаны, которые по особому регламентируются, которые нужно по особому обрабатывать и хранить. И тут речь не о просто ядерном двигателе, а целом буксире, который как будто обрастает вокруг ядерного двигателя. У нас даже Федерацию-Орёл на могут быстро сделать. Всё это удлиняет сроки, но работа ведётся и результаты появятся не сразу, а через 5-10 лет(без гарантий). В этом году функциональный макет должны испытать, где-то в течении пары месяцев. Потом начнут строить на Восточном специальный цех для хранения и испытаний ядерных двигателей, деньги под это уже выделены.
Можешь следить за прогрессом ядерно-космической темы на этих сайтах новостей.
https://www.atomic-energy.ru/NuclearSpace
http://atominfo.ru/index_spacereactor.htm
Аноним 18/02/20 Втр 00:05:55 549554360
>>549540
Что такое функциональный макет?
Аноним 18/02/20 Втр 00:17:26 549556361
>>549554
Макет демонстрирующий некие функции, которые будут представлены в рабочем прототипе и серийном изделии.
Аноним 18/02/20 Втр 00:35:43 549560362
>>549520
Это не так. Ты с белыми дырами спутал, видимо.
Аноним 18/02/20 Втр 00:38:22 549561363
>>549535
>Неизвестно даже, как она гравитационно с материей взаимодействует
Это бы потребовало пересмотра гравитации.
Аноним 18/02/20 Втр 00:39:20 549562364
>>549561
Как что-то плохое.
Аноним 18/02/20 Втр 00:47:01 549563365
>>549562
А есть предпосылки так делать, или это просто маняфантазии?
Мне чет казалось, что большинство считает антиматерь имеющей такую же нормальную массу и так же проваливающейся в гравитационный колодец.
Разве ее не детектили в магнитных поясах?
Аноним 18/02/20 Втр 00:51:47 549564366
>>549563
Ну вообще в коллайдере давно гоняют антипротоны, хз откуда взялись предположения о необычных свойствах
Аноним 18/02/20 Втр 00:53:57 549565367
>>549564
Они там в магнитных ловушках в таких миллипиздрических количествах, что понять - падает оно вниз или вверх - решительно невозможно, потому и дискач. Можно по дефолту предположить что такая же масса и забить хуй, но это не научный подход.
Надо как раз в первую очередь попытаться этой антиматерией опровергнуть существующие теории и тем самым создать новые, от эт по нашему.
Аноним 18/02/20 Втр 09:57:56 549590368
Зачем ракете первая ступень? Привязать к ней 10км гибкий шланг и подавать туда водоврот с кислородом с Земли. Заебенить ба-а-а-шой насос, чтобы давление для подачи на 10км создать не так уж и сложно. Ну или 30км, одна хуйня. Короче, убрать большие баки.

Или так - на ракете только рабочее тело, а греть его до нужных скоростей - электричеством. А электричество - с Земли, ну вы понели, по провуду. Гибкому. 30 км. Из адамантия. Хотя, наверное, и люминь сойдет, если будет с рельсу толщиной. Я тут видел спецпровод для передачи трех гигаватт, с мою ногу толщиной. Три гигаватта по-любому хватит, у ракет меньше мощи обычно.
Аноним 18/02/20 Втр 10:05:24 549592369
>>549590
Светлым головам типа тебя надо тред Омского КБ ракетостроения возрождать.
Аноним 18/02/20 Втр 11:04:11 549599370
>>549565
>падает оно вниз или вверх
Оно падает вниз. Иначе никак.
Аноним 18/02/20 Втр 11:31:18 549604371
>>549590
А теперь пойди и посчитай, сколько твой шланг или провод весить будет.
Аноним 18/02/20 Втр 11:37:00 549605372
>>549604
В космосе же невесомость.
Аноним 18/02/20 Втр 11:41:27 549607373
>>549605
Ты до космоса долети сперва, зеленый.
Аноним 18/02/20 Втр 11:41:42 549608374
>>549599
Сказал как настоящий ученый.
Стоп, нет.
Как вероблядок.
Если есть возможность гипотезу задвинуть - так и надо. Вдруг наши представления о гравитации неверны. Нет догм, нет аксиом, все теории опровержимы.
Аноним 18/02/20 Втр 11:53:33 549609375
>>549565
>Они там в магнитных ловушках в таких миллипиздрических количествах, что понять - падает оно вниз или вверх - решительно невозможно
Разве это не влияло бы на траекторию частицы?
Аноним 18/02/20 Втр 11:54:39 549610376
>>549609
На околосветовых при воздействии йоба-магнитных полей? Слишком незначительно, чтобы заметить.
Аноним 18/02/20 Втр 12:09:49 549611377
Аноним 18/02/20 Втр 12:41:23 549617378
>>547989
А почему бы не сделать первую ступень в виде несущего фюзеляжа и потом сажать как самолет?
Аноним 18/02/20 Втр 14:41:00 549624379
Допустим получится создать термоядерный ракетный двигатель. А как его запускать в космосе? Откуда брать энергию для запуска ядерной реакции синтеза?
Аноним 18/02/20 Втр 17:16:28 549630380
>>549608
Мы бездушно предположим что угодно, наши гипотезы не основаны на здравом смысле, бесцельные опровержения — наша стихия, мы — истинное лицо науки.
Аноним 18/02/20 Втр 17:27:20 549631381
>>549624
Если у тебя есть чудо магниты на 100500Тл, то хоть микроволновкой плазму нагреть. Проблема не нагреть, проблема удержать.

>>547674
Криогенный ГПВРД, ударно-волновой ГПВРД, детонационники. Воздуха наверху мало, хватать еблом его надо быстро. А будешь хватать быстро - либо херящий кпд перегрев на торможении получается, либо сгорать не успевает. Отсюда и сложности.
Ядерная прямоточка ещё есть, если рака яиц не боишься.
Аноним 18/02/20 Втр 19:32:06 549648382
>>549631
Вообще-то нагреть это тоже нихуевая проблема, потому что два из трех способов нагрева плазмы (омический нагрев через протекающий по плазме ток и нагрев электромагнитными волнами) с повышением температуры начинают работать заметно хуже, а сама разогретая плазма начинает очень быстро излучать тепло во все стороны.

Третий способ это прямое закидывание высокоэнергетических ядер в плазму и для этого в ИТЕРе будут размещены два здоровенных инжектора нейтрального луча с разгоняющим напряжением в миллион вольт, каждый размером с автобус и с собственной электрической подстанцией. Хуй знает, где ты в космическом корабле собираешься все это разместить.
Аноним 18/02/20 Втр 19:44:24 549649383
>>549608
Сколько уж раз вы пытались задвигать и всё время обсирались. Пора бы уже принять.
Аноним 18/02/20 Втр 19:47:34 549650384
>>549648
Раз ты нашёл место для охуенного бублика, и для пары автбусов найдёшь
Аноним 18/02/20 Втр 20:59:49 549661385
VY Большого Пса - как это читается? Я имею в виду как произносится гласно? Ученые так друг другу и говорят: "ВЭ ИГРЕК БОЛЬШОГО ПСА"? Или у этой ебанины есть нормальное название? Если нормального названия нет то почему? Разве это не йоба?
Аноним 18/02/20 Втр 21:03:33 549662386
Аноним 18/02/20 Втр 21:04:44 549663387
Аноним 18/02/20 Втр 22:20:39 549671388
>>549649
Так ведь приняли.
Это не делает принятую теорию неопровержимым догматом, есть возможность опровергнуть - только лучше.
Аноним 18/02/20 Втр 22:45:20 549681389
>>549661
Фау-упсилон дес Гроссен Хундс.
Аноним 18/02/20 Втр 22:49:55 549682390
>>549661
ВЫ Большого Пса моего трогали или не вы?
А кроме шуток - да >>549663
>Ви Уай Большого Пса
Вот так.
Ю Уай Щита, кстати.
По-хорошему, наверное, можно уточнить на астрофоруме, но уверен, что единого мнения не будет, лол.
Аноним 18/02/20 Втр 23:07:36 549686391
>>549671
Это не значит также, что нужно про каждое предсказание визжать "врёоти! На самом деле всё не так!". Предсказания стабильно проходят проверку практикой и с наскока такие теории не опровергаются.
Аноним 18/02/20 Втр 23:10:40 549687392
>>549686
>Это не значит также, что нужно про каждое предсказание визжать "врёоти! На самом деле всё не так!".
Нет, я этого и не говорил, с чего ты это запостил вообще?
>Предсказания стабильно проходят проверку практикой и с наскока такие теории не опровергаются.
Разумеется.
Выдвигается новая гипотеза опровергающая рабочую теорию, и в первую очередь опровергается эта гипотеза, а если оказывается, что никто ее не может опровергнуть, то она принимается за рабочую теорию.
Аноним 19/02/20 Срд 00:24:37 549690393
>>549631
>Если у тебя есть чудо магниты на 100500Тл, то хоть микроволновкой плазму нагреть

А откуда брать энергию для "хоть микроволновкой плазму нагреть"?
Аноним 19/02/20 Срд 02:05:43 549697394
Аноним 19/02/20 Срд 05:56:53 549705395
>>549697
>Откуда хочешь.
Так откуда ее взять на космическом корабле? Что, нет ответа?
Аноним 19/02/20 Срд 06:00:20 549706396
>>549705
Выбирай любой источник энергии, наркоман. От появления бублика у тебя другие источники никуда не делись.
Аноним 19/02/20 Срд 06:03:05 549707397
>>549706
>любой источник энергии
Ну так назови откуда взять на космическом корабле энергию для запуска термоядерной реакции в термоядерном ракетном двигателе. назови если знаешь.
Аноним 19/02/20 Срд 06:03:22 549708398
>>549707
Солнечные панели.
Назвал.
Аноним 19/02/20 Срд 06:06:31 549709399
>>549708
>Солнечные панели.
Для разогрева плазмы и для создания магнитного поля нужен, допустим 1 МВт. Ты находишься на орбите Юпитера. Какова площадь солнечных панелей необходимая для получения такой энергии?
Аноним 19/02/20 Срд 06:09:30 549710400
>>549709
А че не на орбите Плутона сразу?
И нахуя солнечные панели на мегаватт мощности, если они тебе один раз нужны только?
Вообще да, панели нахуй, они не нужны.
Лучше сразу беспроводной (микроволновый?) приемник. И зажигать не в ебенях, блядь, а прям сразу после постройки.
Аноним 19/02/20 Срд 06:13:08 549711401
>>549710
>А че не на орбите Плутона сразу?
Т.е. на орбите Юпитера и тем более плутона корабли с термоядерными ракетными двигателями использовать не получится?

>Лучше сразу беспроводной (микроволновый?) приемник
И передавать на орбиту Юпитера энергию с орбиты Земли? А какой мощности нужен передатчик чтобы на орбите Юпитера мощность сигнала была 1 МВт?
Аноним 19/02/20 Срд 06:14:28 549712402
>>549711
>Т.е. на орбите Юпитера и тем более плутона корабли с термоядерными ракетными двигателями использовать не получится?
Нахуя их в ебенях зажигать, наркоман? Сразу зажигай.
>И передавать на орбиту Юпитера энергию с орбиты Земли?
Какого юпитера, блядь, у земли зажигай, дегрод.
Аноним 19/02/20 Срд 06:49:44 549713403
>>549712
> Сразу зажигай.
И ты прилетел к Плутону и такой жгешь топливо на его орбите, да? Лол, ну ты и наивный чукотский юноша.

>Какого юпитера, блядь, у земли зажигай
Т.е. если у твоего корабля по каким-то причинам заглушен двигатель вне орбиты Земли, то тоби пизда? НАЙС.
Аноним 19/02/20 Срд 06:53:54 549714404
>>549713
Ты специально троллишь, или реально тупой?
>И ты прилетел к Плутону и такой жгешь топливо на его орбите, да? Лол, ну ты и наивный чукотский юноша.
"Жгёшь", да. Ты уже ТЕКЁШЬ из треда.
>Т.е. если у твоего корабля по каким-то причинам заглушен двигатель вне орбиты Земли, то тоби пизда? НАЙС.
Чего несёт, блядь.
Аноним 19/02/20 Срд 06:58:47 549715405
>>549714
>"Жгёшь", да.
Т.е. пока все время корабль находится на орбите Плутона и ты занимается своими важными делами типа исследований или чем-то еще то корабль жжет драгоценное топливо и в итоге его не хватает для возвращение? Отличная идея!
>Чего несёт
Ты же сам пишешь, что "завести корабль" можно только на орбите Земли, а это значит если двигатель будет остановлен где-то в космосе, то его уже будет не завести. Охуенный корабль, отлично придумал, бро!
Аноним 19/02/20 Срд 07:04:11 549716406
>>549715
Ага, троллинг тупостью.
Возвращайся с нормальными вопросами, если захочешь конструктив.
Аноним 19/02/20 Срд 07:07:58 549717407
>>549716
>Возвращайся с нормальными вопросами
А что вот эти вопросы

>>549713
>Т.е. если у твоего корабля по каким-то причинам заглушен двигатель вне орбиты Земли, то тоби пизда?

Для тебя "не нормальные"? Может просто ты слишком глуп, а не вопросы?
Аноним 19/02/20 Срд 07:39:06 549720408
Стикер (127Кб, 421x404)
421x404
>>549717
У тебя, по-твоему, надо токамак каждый раз заново зажигать, если надо двигатель задействовать?
Тогда да, пизда, если заглушил.
Ты это хотел услышать?
Нехуй так хуево дизайнить ТЯРД.
Аноним 19/02/20 Срд 10:03:01 549729409
я правильно понимаю, что явление перегрузки в том числе в болиде обуславливается взаимодействием тела человека с поверхностью?
Аноним 19/02/20 Срд 10:24:45 549730410
Скажите пожалуйста, Это правда что лазерами можно на очень ограниченном участке пространства достичь миллиона градусов?
Например кубический микрон?
Какие крутые должны быть лазеры и сколько гигаватт следует вкачать?
Аноним 19/02/20 Срд 10:25:13 549731411
>>549709
>Для разогрева плазмы и для создания магнитного поля нужен, допустим 1 МВт. Ты находишься на орбите Юпитера. Какова площадь солнечных панелей необходимая для получения такой энергии?
Один квадратный метр, лол.
Потому что сверхмощные мегаваттные импульсы идут с конденсаторов. А заряжать их можно хоть от одноваттного источника, просто долго.
Аноним 19/02/20 Срд 10:39:48 549732412
>>549731
>сверхмощные мегаваттные импульсы
удачи с охлаждением в космосе
Аноним 19/02/20 Срд 10:40:24 549733413
Аноним 19/02/20 Срд 10:40:52 549734414
>>549732
У тебя термоядерный реактор, ясен хуй что он без решенного охлаждения не полетит.
Аноним 19/02/20 Срд 11:41:36 549738415
Аноним 19/02/20 Срд 12:42:54 549742416
Аноним 19/02/20 Срд 13:22:21 549745417
Аноним 19/02/20 Срд 15:14:24 549747418
Пиздец конечно, что в космосе даже мегаватт энергии не разместишь. Надо обмануть физику с охлаждением в вакууме, только как? Громоздить квадратные километры радиаторов - не вариант.
Аноним 19/02/20 Срд 15:33:33 549749419
>>549747
нахуй тебе километры шизик
ты формулу забыл что ли
Аноним 19/02/20 Срд 22:13:27 549824420
>>549604
Посчитал - закон Гука даже ддля 50км кабеля еще не нарушается. То есть, сделать-то можно. Вопрос: раз это не исследовали хотя бы и в Омском КБ, значит, что-то принципиально работать не будет. Что? Гигаватт мало?
Аноним 20/02/20 Чтв 00:31:13 549830421
>>549749
Давай поделись формулой, по которой у МКС такие радиаторы.
Аноним 20/02/20 Чтв 01:59:53 549840422
slide-15.jpg (180Кб, 1024x767)
1024x767
Аноним 20/02/20 Чтв 02:03:48 549842423
Насколько близко к с может приближаться скорость материи в аккреционном диске вокруг черной дыры?

Можно ли в дыру упасть отвесно, или закрутит?
Аноним 20/02/20 Чтв 07:42:12 549857424
Если с МКС скинуть провод со штекером и воткнуть его в розетку на земле, МКС намотается на планету и разобьется?
Аноним 20/02/20 Чтв 07:49:12 549858425
>>549857
Провод сгорит не дойдя до земли.
Алсо - при условии что достаточно прочный и не порвется (нет), то затормозит МКС и сведет ее с орбиты.
Аноним 20/02/20 Чтв 09:24:23 549859426
>>549738
не следует ли из этого, что если создать условия, при которых человек в болиде не будет касаться поверхностей, то перегрузки он будет переносить гораздо легче и с наименьшими потерями для здоровья?
Аноним 20/02/20 Чтв 09:25:52 549860427
>>549747
последняя тема - капельное охлаждение вроде.
лично я считаю саму схему бредом, но благодаря грантам хотя бы получены новые уникальные вещества, то есть наука не стоит на месте.
Аноним 20/02/20 Чтв 10:04:16 549864428
>>549840
а теперь ты скажешь какая температура у космического вакуума.
Аноним 20/02/20 Чтв 10:04:39 549865429
Аноним 20/02/20 Чтв 10:05:14 549866430
>>549860
>новые уникальные вещества
капельный холодильник разрабатывался в РФ
что за вещества были получены догадаться не сложно
Аноним 20/02/20 Чтв 10:07:54 549867431
>>549865
намекаешь на то что солнце никак не влияет на эту температуру на орбите Земли?
Аноним 20/02/20 Чтв 10:17:15 549869432
>>549866
>догадаться не сложно
мне не надо догадываться.
я лично знаю людей, которые их синтезировали.
но ты попробуй угадать.
вместе посмеемся.
Аноним 20/02/20 Чтв 10:17:50 549871433
>>549867
В тени - никак не влияет.
Аноним 20/02/20 Чтв 10:18:37 549873434
>>549869
Идеальная водка. Содержит только атомы водорода, кислорода и углерода. Молекулы в идеальных пропорциях, 40.000000х%
Аноним 20/02/20 Чтв 10:21:12 549874435
>>549869
это Рогозинское заведение.
по плодам их узнаете их.
капельный холодильник не будет работать это даже после школьной физики очевидно.
Аноним 20/02/20 Чтв 10:21:20 549875436
>>549873
нет. все сложнее.
свойства следуют их принципа действия установки.
водка там нужна только для того, чтобы придумать.
Аноним 20/02/20 Чтв 10:22:23 549876437
>>549871
В космосе очень мало теней, так что придется тенями пренебречь.
Аноним 20/02/20 Чтв 10:23:27 549877438
>>549876
Ох, мальчик, вот мы идем опять.
Выше было же.
Ничего страшного, всегда можно СДЕЛАТЬ тень.
Аноним 20/02/20 Чтв 10:23:58 549878439
>>549874
С хуев ли не будет? По школьной физике как раз вполне отлично работает.
Аноним 20/02/20 Чтв 10:27:24 549879440
>>549878
а по факту работы прекращены
Аноним 20/02/20 Чтв 10:27:42 549880441
>>549877
>СДЕЛАТЬ тень.
это как?
Аноним 20/02/20 Чтв 10:29:07 549881442
>>549874
>капельный холодильник не будет работать
да нет будет в принципе там ничего сверхестественного.
у меня вопросы возникают другие не к самому принципу, а к надежности системы.
имхо, это все очень зыбко, то, что там на рендерах нарисовали, а в реальности никто не проверял.
чтобы сказать, что работает надо провести пусконаладку, работу до отказа хотя бы разок. а этого наверное даже в планах нету.
Аноним 20/02/20 Чтв 10:31:05 549882443
image.png (619Кб, 777x455)
777x455
image.png (276Кб, 876x556)
876x556
>>549880
Сука, обиженка настучала и информативный пост потерли.

Вот так, как на пикрелейтедах.
Солнечный зонд не вскипает к хуям, криогеника телескопа не выкипает.
Аноним 20/02/20 Чтв 10:49:05 549884444
>>549881
нельзя закладывать в технологию фразу: "ну вы уйдете на новом КК за триллион баксов в тень небесного тела и там остынете...."- за такое денег никто не даст - давай по новой все хуйня!
Аноним 20/02/20 Чтв 10:52:57 549885445
>>549882
это для зондов у которых собственное энерговыделение копеечное.
мы как бы про будущее говорим.
а на данный момент ситуация с физикой такова, что ставить ЯЭДУ на КК невозможно. Хотя сам ЯЭДУ давно готов. А все потому, что охлаждать такую ЙОБУ банально нечем. все упирается в ебаные квадратные километры радиаторов, а на такое денег никто не даст.
Аноним 20/02/20 Чтв 10:56:16 549886446
>>549882
Рассчитывать на криогенику в качестве хладагента это 20 век.
Ну кончится у тебя хладагент и ЧТО ты будешь делать?
Аноним 20/02/20 Чтв 10:59:55 549887447
>>549885
>это для зондов у которых собственное энерговыделение копеечное.
Ну да, вот у них и небольшие панельки/хладагент.
>мы как бы про будущее говорим.
Увеличишь тепловыделение - увеличь радиаторы, в чем проблема? Щит пошире, и все, у тебя конус умбры огромный.
>а на данный момент ситуация с физикой такова, что ставить ЯЭДУ на КК невозможно. Хотя сам ЯЭДУ давно готов. А все потому, что охлаждать такую ЙОБУ банально нечем. все упирается в ебаные квадратные километры радиаторов, а на такое денег никто не даст.
Все так.
А еще нетзадач.

>>549886
>Ну кончится у тебя хладагент и ЧТО ты будешь делать?
Заканчивать миссию, хули. Хладагента как раз заложили на время миссии.
В теории он может подослепнуть и работать без хладагента наверное в продленной миссии? Кто знает?
Аноним 20/02/20 Чтв 11:12:27 549888448
А что там за волшебные материалы создали рогозинцы?
Аноним 20/02/20 Чтв 11:15:21 549889449
>>549888
Жижа которая хуево в вакууме испаряется, скорей всего.
И как мне послышалось, основная проблема в том, что на этапе улавливания она расплескивается нахоой.

Инсайдер, опровергни.
Аноним 20/02/20 Чтв 11:29:50 549890450
>>549889
это техническая проблема
а они обосрались с научной
Аноним 20/02/20 Чтв 11:32:03 549891451
>>549890
Конкретики, я так понимаю, ждать от тебя нет смысла.
Аноним 20/02/20 Чтв 11:38:49 549892452
>>549888
>рогозинцы
ничего не создали.
изначально то как раз капельный холодильник разрабатывался для полета к марсу.
потом пришел рогозин и деньги перенаправились в другое русло.
на Луну и немношк в шубохранилище рогозина
Аноним 20/02/20 Чтв 11:40:11 549893453
>>549890
это далеко не техническая проблема - то о чем этот анон говорит
>Жижа которая хуево в вакууме испаряется
и на этом этапе вроде никто не обосрался.
а о каком этапе ты говоришь я не знаю.
Аноним 20/02/20 Чтв 11:48:59 549895454
>>549893
том что эффективная поверхность рассеивания априори меньше чем у простого радиатора
Аноним 20/02/20 Чтв 11:49:57 549896455
fukken lold spc.jpg (176Кб, 640x400)
640x400
>>549895
Ебааааааааааааать, как же я проиграл!
Воистину, нет дна у мелкобуквы.
Аноним 20/02/20 Чтв 11:56:35 549897456
>>549895
>том что эффективная поверхность рассеивания априори меньше чем у простого радиатора
там дело не в поверхности так то, а в бесплатно и без смс
Аноним 20/02/20 Чтв 12:00:22 549898457
>>549824
Ну во первых, шланг весит и поднимать его будет ракета. 50 километров шланга будут весить дохуя.
Во вторых шланг будет лететь вместе с ракетой, с нихуевой скоростью. И скорость течения топлива должна будет быть больше. Удачи загонять топливо в трубу со скоростью в 300-500-1000 метров в секунду.
В третьих, что бы качать топливо вверх тебе нужно будет преодолевать давление столба. Скажем каждые 10 метров воды будут обеспечивать давление в 1 атмосферу. Керосин имеет плотность чуть ниже, но уже 10 км столб керосина будет давать ~820 атмосфер. Удачи найти такой насос, который способен такое давление преодлолеть и найти такой шланг, который таким давлением не разорвет.
Аноним 20/02/20 Чтв 14:33:04 549917458
>>549590
>Привязать к ней 10км гибкий шланг и подавать туда водоврот с кислородом с Земли. Заебенить ба-а-а-шой насос, чтобы давление для подачи на 10км создать не так уж и сложно. Ну или 30км, одна хуйня. Короче, убрать большие баки.
Было уже в омском КБ.
Принципиальная проблема в том, что эффект Оберта работает в обратную сторону: чем быстрее ты летишь - тем больше рабочего тела приходится подавать. Энергетически нихуя не оптимально получается: в результате ты сожжёшь намного больше, чем сжёг бы в обычной первой ступени.

>Или так - на ракете только рабочее тело, а греть его до нужных скоростей - электричеством. А электричество - с Земли, ну вы понели, по провуду.
А вот эту тему я копал. Если кратко, то всё упирается в совершенно чудовищный вес кабеля, с которым даже при УИ в тысячи секунд массы для ПН просто не остаётся.
Уменьшить вес кабеля можно за счёт роста вольтажа (для омического нагрева так даже лучше), но оптимальный режим начинается от десятков мегавольт, вплоть до гигавольт. И хотя мегавольтные ЛЭП в совке существовали, в случае с около-гигавольтными уже начинаются большие вопросы, как это протягивать, чтобы не коротнуло. Ведь изоляция у нас строго воздушная - иначе масса изолятора съедает все преимущества.

Теоретически решение я вижу одно: передача энергии по одному проводу. Корпус ракеты это однополюсный конденсатор очень малой ёмкости и условно-неограниченного вольтажа. Подавая на него гигавольтное переменное напряжение с земли мы получим ебическую мощность в тонком кабеле, достаточную, чтобы между кабелем и корпусом горела дуга в десятки тысяч градусов, которая и будет греть рабочее тело с высоким УИ.
Аноним 20/02/20 Чтв 14:52:11 549919459
>>549599
>Оно падает вниз. Иначе никак.
Видишь ли какая хуйня. Если антиматерия падает вниз, то CPT-инвариантность приводит к очень странной трактовке второго начала термодинамики, ибо в этом случае с точки зрения антиматерии энтропия для антиматерии всегда падает. Какие эффекты это может повлечь на крупномасштабных системах - хрен его знает, но мало не покажется.
Если антиматерия падает вверх, то у нас тут во-первых ололо антигравитация, во-вторых море неясности по эквивалентности гравитационной и инерционной массы.
Аноним 20/02/20 Чтв 15:24:23 549923460
Где можно безвозмездно посраться со всякими плоскоземельцами, рептилоидами и прочими эфирами?
Аноним 20/02/20 Чтв 15:43:38 549929461
>>549919
>Если антиматерия падает вниз, то CPT-инвариантность приводит к очень странной трактовке второго начала термодинамики, ибо в этом случае с точки зрения антиматерии энтропия для антиматерии всегда падает.

При обращении времени термодинамика начинает работать в обратную сторону, кто бы мог подумать! Гораздо страннее бы было, если бы энтропия не убывала, тогда бы проблемы начались уже в настоящем направлении времени.
Аноним 20/02/20 Чтв 16:10:11 549933462
Какие есть пруфы наличия Гагарина внутри корабля Восток 1 во время его полета?
Аноним 20/02/20 Чтв 16:48:20 549938463
>>549933
Очевидцы, фото, отчёты и прочие бумаги, отсутствие каких либо доказательств даже косвенных о противоположном, практическая невозможность скрыть столь масштабный проект, остаточная радиация на теле сабжа.
Всё указывает на то, что он всё же был там.
Аноним 20/02/20 Чтв 16:54:01 549940464
>>549896
хорошо тебе проигрывать в своем манямирке с манятеориями. попили бабало на принципиально бессмысленном холодильнике который никому не нужен и бегом на двач маняпроигрывать
Аноним 20/02/20 Чтв 16:54:20 549941465
Аноним 20/02/20 Чтв 16:58:06 549942466
>>549923
Временами прямо здесь.
Аноним 20/02/20 Чтв 17:10:38 549943467
Аноним 20/02/20 Чтв 17:29:06 549946468
>>549943
Я там был пару лет назад и там уже было тухло и большинство сообщений точно такие же охотники на рептилоидов.
Аноним 20/02/20 Чтв 17:33:47 549947469
>>549941
Есть архивы смотри. Я не особый эксперт по поиску.
Вроде на УМП ещё подобный миф разбирали.
Аноним 20/02/20 Чтв 19:21:13 549953470
1rm87bel3dm31.jpg (318Кб, 1080x1350)
1080x1350
Дайте фото если есть или инфу ПО человеческим отходам (не говно) ногти, кожа, волосы - за период пребывания на станции. Интересно глянуть на это
Аноним 20/02/20 Чтв 19:48:19 549958471
>>549953
А что их смотреть в пакет положил и с бытовым мусором выкинул, а волосы в фильтре системы филтрации сточных вод. А кожа в системе фильтрации воздуха.
Имхо. Чисто гипотеза.
Аноним 20/02/20 Чтв 21:10:08 549960472
>>549958
Это было бы хорошее наглядное подтверждение факту об ограничении клеточных делений. С человека ведь постоянно отшелушивается мёртвая кожа, вот её количество за год думаю произвело бы впечатление.
Аноним 20/02/20 Чтв 21:31:32 549961473
>>549898
Рукава СВД под 3000 атмосфер имеют массу от 100 до 1500 грамм на метр длины. Несколько тонн батут с собой утащит, не страшно.
Насосы СВД бывают до 5000 атмосфер - ими на больших скоростях вытесняющие жидкости в скважины закачивают и гоморазрывы пластов устраивают. То есть, и большую скорость подачи тоже уже освоили. Кстати, а если насос на земле эти 3000 бар делает, так ли нужен дорогущий ТНА на ракете?
Еще контраргументы?
Аноним 20/02/20 Чтв 21:41:08 549962474
>>549859
Нет, тебе нужно создать условия, при котором при перегрузке на человека не будет действовать сила реакции опоры.
Аноним 20/02/20 Чтв 21:58:54 549964475
>>549960
Я не думаю что у них фильтры так редко меняют. Еженедельное ПХД никто не отменял. А за неделю что с тебя обсыпеться?
Аноним 20/02/20 Чтв 22:35:46 549968476
>>549917
>совершенно чудовищный вес кабеля
Современный изолированный ПТФЭ 750кВ кабель на 2А, скажем, суб-5кг/метр. То есть, 30 км такого кабеля - треть от сухой массы первой ступени батута. Без топлива. Короче, кабели есть. Можно ли запустить ракету на голом искричетстве - что и как лучше всего нагревать?
Аноним 20/02/20 Чтв 23:10:17 549973477
>>549968
>на 2А
Это вообще ни о чем, всего 1,5 МВт мощности. На то, чтобы создать тягу, эквивалентную какому-нибудь спейсиксовскому Мерлину (~300 кг топлива в секунду на скорости в ~3000 м/с) уйдет больше гигаватта мощности, на 9 двигателей — больше 10 ГВт.

И вот уже метр кабеля весит не 5 кг, а неподъемные 30 тонн, не говоря уже о гигантской электростанции, которую придется построить на космодроме.
Аноним 20/02/20 Чтв 23:47:22 549981478
>>549919
>Если антиматерия падает вниз, то CPT-инвариантность приводит к очень странной трактовке второго начала термодинамики, ибо в этом случае с точки зрения антиматерии энтропия для антиматерии всегда падает.
Тут можно поподробнее?
Аноним 20/02/20 Чтв 23:49:39 549983479
>>549923
В /b/ иногда бывают плоскоземельные треды, но дискач там невозможен. Не потому, что /b/, а потому, что плоскоземельщики либо тролли, либо отбитые наглухо дегенераты, с которыми априори нормальный диалог невозможен.
На плеббите даже не пытайся, банят за обычные вопросы.
На тытрубе либо игнор, либо переход на личности.
На фейсбуке тоже.

Среди этих ебанутых есть только единицы не отбитых, а случайно примкнувших сообразительных, которым хорошо голову запудрили, но ты хуй отыщешь таких.
Аноним 20/02/20 Чтв 23:53:48 549985480
>>549958
Каких еще сточных вод? Ты что, думаешь они там ДУШ принимают что ли? Душей в космосе давно не было.

>>549953
Волосы стригут под пылесосом, все в пылесборник заходит. Выпавшие остаются на сетках впускных решеток вентиляции.
Стригут ногти у тех же решеток.
Кожа тоже там.
Потом эти сетки пылесосят.

>Интересно глянуть на это
https://youtu.be/xICkLB3vAeU
Аноним 20/02/20 Чтв 23:55:08 549986481
>>549961
>гоморазрывы
Почему-то проиграл.

Чето мне сдается, ты хуйню заливаешь, и все не так просто.
Аноним 20/02/20 Чтв 23:55:35 549987482
>>549962
Просто будь снаружи разгоняющегося корабля.
Аноним 21/02/20 Птн 00:53:53 550004483
>>549729
есть еще собственная инерция, и на больших ускорениях она размажет или разорвет твое тело без всяких поверхностей
Аноним 21/02/20 Птн 01:25:12 550013484
Сап двач, я нихуя не умею, но хочу оформить себе новое хобби - кодинг и орбитальную механику. Совместить приятное с полезным. Идея в том, чтобы создать в браузере симулятор солнечной системы с пусками космических аппаратов, как в кербале но только с обзорным видом, когда настраиваешь маневр и орбиту.

Какую литературу посоветуете почитать, чтобы потом эту теорию можно было воплотить в алгоритм?
Аноним 21/02/20 Птн 01:26:17 550014485
>>550013
Левантовский, хули, платина.
Аноним 21/02/20 Птн 01:30:23 550015486
>>549964
Ьоя мне нужна СУММА отходов человека или группы людей в закрытом пространстве за длительный период как нахуй горы башмаков от сожжёных в нацистских печах - че блядь непонятного. Нахрен мне твой усатый пидор, я знаю как сттригрутся ногти ёбанарот
Аноним 21/02/20 Птн 01:30:43 550017487
Аноним 21/02/20 Птн 01:32:59 550019488
>>550015
Ну епт, сам спроси их, если тебе так надо, здесь никто таким вопросом не задавался.
Аноним 21/02/20 Птн 01:46:58 550021489
5o75og96gx041.jpg (336Кб, 1108x1478)
1108x1478
Аноним 21/02/20 Птн 02:47:58 550027490
>>550014
Благодарю, почитаю обязательно. Алсо я сейчас смотрю на ютубе ролики про орбитальную механику и оказывается, что во всех этих кербалах решается только система двух тел и вокруг солнца вращается планета, а не барицентр планеты + спутников. То есть до сих пор точного решения не существует?
Аноним 21/02/20 Птн 02:50:13 550028491
>>550027
Аналитического решения не существует.
Только итеративные подсчеты.
В Universe Sandbox все тела гравитируют и влияют друг на друга, но если выставишь большой временной интервал - точность уплывет к хуям и систему распидорасит.
Аноним 21/02/20 Птн 02:53:00 550029492
>>550028
>Universe Sandbox
Пиздос, оказывается уже все придумали до меня... Это как раз что что я хочу сделать.
Аноним 21/02/20 Птн 02:54:51 550030493
>>550029
Не, ты же в браузере хотел.
Вот тебе менее функциональный, но удобный инструмент в браузере https://www.solarsystemscope.com
Аноним 21/02/20 Птн 02:56:35 550031494
>>550030
Ну вот зачем ты меня так расстроил, что это тебе дало?

Ну и ладно, я все равно хотел выучить как работают орбиты, поэтому буду пробовать только для себя.
Аноним 21/02/20 Птн 02:58:09 550032495
>>550031
Так ебашь, это не повод сдаваться.
Наоборот - удобно подглядеть получится. А со своей перспективой сможешь чего нового привнести в теории.
Аноним 21/02/20 Птн 14:55:16 550061496
Поясните, если на большой глубине очень сильное давление что даже лодки из металла имплодят. Как там животные на дне марианской впадине существуют? Ведь у них должны быть стальная кожа.
Аноним 21/02/20 Птн 15:00:47 550062497
image.png (266Кб, 533x390)
533x390
>>550061
Не должна.
Подлодки имплодит потому, что внутри пустота, там всего-ничего давления по сравнению с наружным.
А у живности внутри давление сопоставимо с окружающим, так что сдавливаться нечему.
Зато если вытащишь наружу - их раскукожит внутренним давлением.
Аноним 21/02/20 Птн 15:02:59 550063498
>>550061
У лодки из металла проблема в том, что внутри нее воздух под гораздо меньшим давлением, чем у воды снаружи. У животных такой проблемы нет: у них внутри всякие водные растворы ровно с забортным давлением (плавательные пузыри есть только у мелководных рыб, например). Пидорасить может разве что отдельные макромолекулы (и в гораздо меньшей степени), что зависит от их структуры. Кстати, у глубоководных организмов некоторые белки в метаболизме другие, потому что "обычные" либо разваливаются, либо хреново работают при большом давлении.
Аноним 21/02/20 Птн 16:10:39 550071499
>>549947
Ясно, значит пруфов нет.
Аноним 21/02/20 Птн 16:49:05 550075500
>>550071
Да, это все очевидное наебалово
Аноним 21/02/20 Птн 19:21:42 550091501
>>550075
Какие вообще есть пруфы наличия космоса?
Аноним 21/02/20 Птн 19:42:06 550093502
>>550071
Нельзя добыть пруф отсутствия чего-либо. У тебя есть фото\видео\док архивы чисти, чисти, чисти. Это ты должен искать то, что там был маникен. В противном случае нулевая гипотеза.
Это из разряда: "дайте пруфы, что твой пра-дедушка не рептилоид."
>>549985
Голову они всё же моют и сливают пену. И засасывают это сливным отверстием или как ты выразился пылесосом.
>>550091
Исходя из определения - космос это всё. Мы все сейчас в космосе. Ты имеешь ввиду безвоздушное пространство? Так в любой барокамере такое есть. Или ты хочешь пруф что с набором высоты падает плотность воздуха? Так заберись в горы. Пруф чего ты хочеш?
Аноним 21/02/20 Птн 21:06:28 550099503
>>550093
>Пруф чего ты хочеш?
Наличия чего-то за небом.
Аноним 21/02/20 Птн 21:39:38 550106504
>>549986
Ну так я и спрашиваю, что может быть не так просто. Ведь технологии есть, по крайней мере тут адамантия не требуется. А раз есть - значит об этом думали. Раз не придумали, значит что-то помешало концептуально. Вот и интересно, что. Узнаем уже в другом треде
Аноним 21/02/20 Птн 21:52:03 550107505
>>550099
За небом и нет ничего. В небе и есть безвоздушное пространство и т.д. А "за небом" - это уже вопрос к конечности вселенной
Аноним 21/02/20 Птн 22:16:09 550108506
>>550099
Что ты подразумеваешь под небом? Что то я не вижу никакого потолка.
Аноним 21/02/20 Птн 22:21:16 550109507
>>550061
>>550062
>>550063
Всё это конечно справедливо для глубин меньше 8к. Но к 11к уже заметные трудности. В частности 11к замечен лишь 1 вид живущих там многоклеточных и это рачок. Для хордовых это считается слабо-возможным.
Аноним 21/02/20 Птн 22:35:24 550110508
>>550109
Но это не от того, что их там физически пидорасить будет. А от того, что при больших давлениях химические реакции идут по-другому, и просто так перенести "обычный" метаболизм на предельные глубины не получится. С хордовыми проблемы, скорее всего, потому, что они банально сложнее устроены. И чтобы они могли так глубоко жить, нужно перестроить больше метаболических циклов.
Аноним 21/02/20 Птн 23:20:09 550115509
Прошу пояснить за dyna soar( а конкретнее про его тепло защиту).
Почему такая схема "не взлетела", и далее на различных многоразовых аппаратах применяется плитка?
Аноним 21/02/20 Птн 23:29:41 550117510
Откуда берётся гравитация нахуй?
Аноним 21/02/20 Птн 23:32:19 550118511
>>549985
А дрочат куда? Тоже в вентиляцию?
Аноним 21/02/20 Птн 23:45:34 550120512
Аноним 21/02/20 Птн 23:50:03 550121513
>>550117
Гравитации не существует, это кажущаяся сила, вызываемая искривлением пространства.
Аноним 22/02/20 Суб 00:06:31 550122514
>>550121
В смысле кажущаяся? На ней всё основано.
Аноним 22/02/20 Суб 01:09:20 550127515
так я и не понял сколько мегаватт надо потратить на лазоры что бы они разогрели 1 кубический микрон до миллиона градусов хотя бы на секунду
Аноним 22/02/20 Суб 01:19:46 550128516
>>550118
В салфеточку, блжад, ты че, на земле на пол дрочишь и ждешь пока мамка вытрет?
Аноним 22/02/20 Суб 01:20:37 550129517
>>550127
Кубический микрон чего?
В чем проблема посчитать? Физика 6 класс.
Аноним 22/02/20 Суб 01:21:13 550130518
Аноним 22/02/20 Суб 01:39:58 550131519
>>550129
я не инженер, я пиарщик
Аноним 22/02/20 Суб 02:03:38 550134520
>>550117
Так как это НЁХ которую не пощупать не потрогать в ход могут идти лишь предположения.
Сейчас самая популярная следующая.
Гравитация это иллюзия из-за искривления пространства. Само по себе падение это прямолинейное движение, но лишь в 4х мерном пространстве. В трёхмерном же это получается дуга с замедлением в апогее.
Аноним 22/02/20 Суб 14:05:41 550168521
>>547634 (OP)
У кого-нибудь есть карта плотности астероидов солнечной системы?
Аноним 22/02/20 Суб 14:22:11 550170522
Аноним 27/02/20 Чтв 10:41:46 550916523
>>549523
аннигиляция это взаимоуничтожение?
Настройки X
Ответить в тред X
15000 [S]
Макс объем: 40Mб, макс кол-во файлов: 4
Кликни/брось файл/ctrl-v
Стикеры X
Избранное / Топ тредов